Test-1 GS(Paper-I) Detailed Solution

You might also like

Download as pdf or txt
Download as pdf or txt
You are on page 1of 76

ALL INDIA OPEN MOCK TEST - 1 - SOLUTION

Q1. Consider the following statements with reference to Ajanta Cave paintings:
1. The walls of the caves have both murals and fresco paintings.
2. They portray human values and social fabric along with styles, costumes and ornaments of that period.
3. They mostly belong to the Mauryan period.
4. They are predominantly related to Buddhism and Jainism.
5. Vajrapani, Manjusri and Padmapani are a few examples of cave paintings of Ajanta.
How many of the above statements are correct?
A. Only two
B. Only three
C. Only four
D. All five
Answer: B
Explanation:
 Statement 1 is correct: The Ajanta Caves are 30 rock-cut Buddhist cave monuments dating from the 2nd century
BCE to about 6th century CE in the Aurangabad district of Maharashtra. The walls of the caves have both murals
and fresco paintings (painted on wet plaster). They use tempera style, i.e. use of pigments.
 Statement 2 is correct: The paintings found in the Ajanta Caves provide a fascinating glimpse into ancient Indian
society, culture, and values. These paintings showcase various aspects of life during that period, including
depictions of royalty, court scenes, everyday life, and religious themes. The art in Ajanta not only displays the
aesthetic styles prevalent at the time but also offers insights into social hierarchies, religious practices, and
cultural exchanges.
 Statement 3 is not correct: The Ajanta cave paintings are primarily associated with the Gupta period, which
lasted from around 320 to 550 CE in India. During the Gupta period, India saw a flourishing of art, architecture,
literature, and science. The Ajanta cave paintings are considered masterpieces of Indian art and are renowned
for their exquisite beauty and skillful execution.
o The Vakataka dynasty, which ruled over the region where the Ajanta caves are located, was also a major
patron of the arts and is credited with commissioning many of the paintings and sculptures found in the
caves.
 Statement 4 is not correct: The paintings of Ajanta Caves are predominantly related to Buddhism, with some
elements of Hinduism, but not Jainism. The paintings in the Ajanta Caves are primarily Buddhist in subject matter,
focusing on the life of Buddha and Jataka stories, which are Buddhist legends about the Buddha's previous births.
 Statement 5 is correct: Paintings of various Bodhisattvas in tribhanga pose in Cave No. 1: Vajrapani (protector
and guide, a symbol of Buddha’s power), Manjusri (manifestation of Buddha’s wisdom) and Padmapani
(Avalokitesvara) (symbol of Buddha’s compassion).
Hence, option B is correct.

2nd Floor, 45 Pusa Road, Opp. Metro Pillar 128, Karol Bagh, New Delhi-110005
Ph: 08045248491, 7041021151 | Email: students@levelupias.com
Q2. Consider the following statements with reference to the Nagara style of temple architecture:
1. The entire temple often built on a stone platform with steps leading up to it.
2. It is characterised by elaborate gateways or gopurams.
3. The garbhagriha is located directly under the tallest tower.
How many of the above statements is/are NOT correct?
A. Only one
B. Only two
C. All three
D. None
Answer: A
Explanation:
 Statement 1 is correct: A Nagara-style temple often built
on a stone platform with steps leading up to it. The
temple is typically constructed on a raised platform
known as a "jagati," which provides a base for the temple
structure. This platform can vary in height and size
depending on the design and the terrain of the temple
site.
 Statement 2 is not correct: Nagara-style temples, which
are prevalent in North India, typically do not feature
elaborate boundary walls or gateways as commonly seen
in South Indian temple architecture.
o In contrast to the Dravidian-style temples of
South India, which often have towering
gopurams (ornate entrance gateways) and
compound walls that enclose the temple
complex, Nagara-style temples usually have a
simpler layout.
 Statement 3 is correct: The garbhagriha, or sanctum, is
always located directly below the tallest tower, or
shikhara, in a Nagara temple. The garbhagriha is the
most important part of some of the earliest temples and
is where the image of the chief deity is installed.
Hence, option A is correct.

2nd Floor, 45 Pusa Road, Opp. Metro Pillar 128, Karol Bagh, New Delhi-110005
Ph: 08045248491, 7041021151 | Email: students@levelupias.com
Q3. Which among the following events are part of the sculptures at Mahabalipuram?
1. Krishna lifting the Govardhan Mountain
2. Ravana Shaking Mount Kailash
3. The great battle between the goddess Durga and the buffalo demon.
4. Vishnu rescued Mother Earth from the depths of the cosmic ocean.
Select the correct option using the code given below:
A. 1, 2 and 3only
B. 2, 3 and 4 only
C. 1, 2 and 4 only
D. 1, 3 and 4 only
Answer: D
Explanation:
The Mahabalipuram group of monuments comprises rock-cut cave temples, monolithic temples, bas-relief sculptures,
structural temples, and the excavated remains of temples. These impressive structures were created by the Pallava
dynasty, which governed the region from the 6th to the 9th centuries CE. The group of monuments includes:
 Statement 1 is correct: This event is depicted in the Krishna Mandapa, which is located next to Arjuna's Penance
at Mahabalipuram. The relief shows Krishna lifting the Govardhan Mountain with one hand to protect the
villagers from a storm sent by Indra, the god of rain. This episode from Hindu mythology symbolizes Krishna's
compassion and his ability to protect his devotees.
 Statement 2 is not correct: Ravana Shaking Mount Kailash is not part of the Group of Monuments at
Mahabalipuram. Instead, it is depicted in the Ellora caves, which are located in Maharashtra. The depiction shows
Ravana, the demon king from the epic Ramayana, attempting to shake Mount Kailash, the abode of Lord Shiva.
This act was to prove his power and challenge the might of Lord Shiva.
 Statement 3 is correct: The great battle between the goddess Durga and the buffalo demon is depicted in the
Mahishasuramardini cave at Mahabalipuram. The relief shows the goddess Durga in her fierce form, battling
against the buffalo demon Mahishasura. This episode from Hindu mythology symbolizes the triumph of good
over evil and the power of the divine feminine.
 Statement 4 is correct: Vishnu rescued Mother Earth from the depths of the cosmic ocean is depicted in the
Adivaraha cave at Mahabalipuram. The relief shows Vishnu in his Varaha avatar, a form where he appears as a
boar. Vishnu rescues Mother Earth, represented as a female figure, from the depths of the cosmic ocean after she
was abducted by the demon Hiranyaksha. This event symbolizes Vishnu's role as the protector of the universe
and his compassion towards all beings.
Hence, option D is correct.

2nd Floor, 45 Pusa Road, Opp. Metro Pillar 128, Karol Bagh, New Delhi-110005
Ph: 08045248491, 7041021151 | Email: students@levelupias.com
Q4. Which of the following are correctly matched?
1. Rajatarangini – History of Kashmir
2. Tuti Nama – A love story between Muhammad Bin Tughlaq and his Queen
3. Nagananda – Prince Jimutavahana and the divine Garuda
4. Malavikagnimitram – Romantic tale of a Shunga prince
Select the correct answer using the code given below
A. 1 and 2 only
B. 1, 3 and 4 only
C. 1, 2 and 4 only
D. 3 and 4 only
Answer: B
Explanation:
 Pair 1 is correct: Kalhana, a Kashmiri Brahmin, authored the chronicle Rajatarangini, meaning 'the river of kings,'
in Sanskrit. This historical work provides a comprehensive account of Kashmir and northwest India. It is
comprised of 7826 verses and is divided into eight books known as Tarangas, or 'waves,' making it the earliest
historical source of its kind.
 Pair 2 is not correct: "Tutinama" is a collection of 52 stories in Persian commissioned and illustrated by the
Mughal Emperor Akbar. Authored by Ziya' al-Din Nakhshabi, it revolves around a parrot narrating tales to its
owner Khujasta over 52 nights. The story reveals that Khujasta's husband, Maimunis, a merchant, left her in the
company of a mynah and a parrot while he was away on business.
 Pair 3 is correct: Harshavardhana, the emperor of the Vardhana dynasty who ruled from Kannauj, is credited with
writing the play Nagananda. This Sanskrit play revolves around Emperor Harsha and tells the story of Prince
Jimutavahana, who sacrifices his own body to prevent the sacrifice of serpents to the divine Garuda.
 Pair 4 is correct: "Malavikagnimitram, a drama penned by Kalidasa in the 5th century BC, narrates the love story
of Agnimitra, the Shunga Emperor in Vidisha, and his affection for a maid serving his chief Queen. His infatuation
is sparked by a painting of Malavika, an exiled servant girl, leading to a romantic tale centered around Agnimitra,
the prince of the Sunga dynasty."
Hence, option B is correct.

Q5. Consider the following statements about the society of Early Vedic Period:
1. Early Vedic Society was divided on Caste lines.
2. Warrior category was referred as ‘Rajanya’.
Which of the above statement(s) is/are correct?
A. 1 only
B. 2 only
C. Both 1 and 2
D. Neither 1 nor 2

Answer: B

2nd Floor, 45 Pusa Road, Opp. Metro Pillar 128, Karol Bagh, New Delhi-110005
Ph: 08045248491, 7041021151 | Email: students@levelupias.com
Explanation:
 Statement 1 is not correct: During the Early Vedic Period, society in the Indian subcontinent was primarily
organized along kinship lines rather than the rigid caste system that later developed.
o Social organization was based on kinship ties, clan networks, and tribal affiliations. People belonged to
different clans (Vis) which formed tribes (Jana). These tribes were led by chiefs (Raja or Gopati).
 Statement 2 is correct: In Early Vedic Society, the warrior category was known as 'Rajanya'. The Rajanyas were
the ruling class and played a crucial role in society as protectors and leaders in times of war.
o The term 'Rajanya' is derived from the Sanskrit word 'raja', which means king or ruler. The Rajanyas held
significant power and authority within their tribes or communities, often serving as chiefs or kings. They
were responsible for leading military expeditions, protecting the tribe from external threats, and
maintaining order within the community.
Hence, option B is correct.

Q6. Consider the following statements:


Statement-I:
Turkish advent in India made true arches popular in Indian buildings.
Statement-II:
Lime Mortar was brought by the immigrant Muslims during the Delhi Sultanate rule.
Which one of the following is correct in respect of the above statements?
A. Both Statement-I and Statement-II are correct and Statement-II is the correct explanation for Statement-I.
B. Both Statement-I and Statement-II are correct and Statement-II is not the correct explanation for Statement-I.
C. Statement-I is correct but Statement-II is incorrect.
D. Statement-I is incorrect but Statement-II is correct.
Answer: A
Explanation:
 Statement-I is correct: Before the arrival of Turks, Indian architecture predominantly utilized post-and-beam
construction methods. While arches existed, they were often corbelled arches, limiting their size and strength.
However, the arrival of the Turks in India during the Delhi Sultanate period did popularize the use of true arches
in Indian buildings.
 Statement-II is correct: Lime mortar, a superior binding material for constructing arches, was widely adopted
during the Delhi Sultanate era. Lime mortar, made from burnt limestone and an aggregate like sand, offered
several advantages over traditional binding materials like mud. Its widespread use for constructing true arches
became characteristic of the Turkish era. They brought expertise in the use of lime mortar, which played a
crucial role in the construction of larger and more permanent arches, contributing to the architectural
achievements of the Delhi Sultanate period.
Hence, option A is correct.

2nd Floor, 45 Pusa Road, Opp. Metro Pillar 128, Karol Bagh, New Delhi-110005
Ph: 08045248491, 7041021151 | Email: students@levelupias.com
Q7. Consider the following statements with reference to the Swadeshi Movement:
1. Sanjivani newspaper suggested the idea of boycotting British goods during the Swadeshi Movement.
2. The Carlyle Circular was issued to keep students away from the Swadeshi Movement.
3. The movement witnessed huge support from the peasantry.
4. This movement led to the revival and emergence of nationalist art and literature.
How many of the above statements is/are correct?
A. Only one
B. Only two
C. Only three
D. All four
Answer: C
Explanation:
Swadeshi Movement was launched in 1905 as a protest against the partition of Bengal. The movement was the
steppingstone of the Nationalist movement, which led to the beginning of an organized political movement in India.
 Statement 1 is correct: The Sanjivani newspaper, edited by Krishnakumar Mitra, played a crucial role in
advocating for the boycott of British goods. It suggested the idea of boycotting British goods as a means of protest
against British colonial rule and exploitation. This suggestion gained traction and eventually became a central
aspect of the Swadeshi Movement.
 Statement 2 is correct: The Carlyle Circular was issued by the British authorities in response to the participation
of students in the Swadeshi Movement. It threatened students with the withdrawal of grants, scholarships, and
affiliations of the institutions they belonged to if they continued to participate in the movement. This was a
tactic employed by the British to suppress dissent and discourage students from supporting nationalist causes.
 Statement 3 is not correct: Swadeshi Movement did not witness significant support from the peasantry. The
movement primarily attracted support from the upper and middle classes, intellectuals, and some sections of
the urban population. The peasantry, which constituted a significant portion of the Indian population, remained
largely unaffected by the movement.
o Issues pertinent to the peasantry, such as agrarian distress, debt burdens, and land rights, were not
adequately addressed by the Swadeshi Movement, resulting in limited participation from rural
communities.
 Statement 4 is correct: The Swadeshi Movement fostered the revival and emergence of nationalist art and
literature in India. Artists, writers, poets, and intellectuals actively participated in the movement and used various
mediums to express nationalist sentiments. This period saw the flourishing of nationalist literature, paintings,
songs, and cultural expressions that celebrated Indian identity, heritage, and resistance against British
colonialism. The movement inspired a cultural renaissance and contributed to the development of a distinct
Indian nationalist consciousness.
Hence, option C is correct.

2nd Floor, 45 Pusa Road, Opp. Metro Pillar 128, Karol Bagh, New Delhi-110005
Ph: 08045248491, 7041021151 | Email: students@levelupias.com
Q8. Consider the following statements:
1. He was associated with the Justice Party and the Self Respect Movement.
2. He was involved in the Vaikom Satyagraha of 1924.
3. He launched Dravidar Kazhagam.
4. He focused on social, cultural and gender inequalities, and his reform agenda questioned matters of faith, gender and
tradition.
The above statements refer to which of the following personalities?
A. E. V. Ramasamy Naicker
B. V. O. Chidambaram Pillai
C. S. Subramania Iyer
D. C. Rajagopalachari
Answer: A
Explanation:
 E.V. Ramasamy Naicker (1879–1973), better known as “Periyar” (literally “the big man”), was a firebrand Indian
social reformer and politician. He is considered the "Father of the Dravidian movement," which advocated for
social justice and equality for non-Brahmin castes in South India.
 Periyar is remembered for the Self Respect Movement to redeem the identity and self-respect of Tamils. He
envisaged a Dravida homeland of Dravida Nadu, and launched a political party, Dravidar Kazhagam (DK). He was
also associated with the Justice Party.
 His fame spread beyond the Tamil region during the Vaikom Satyagraha of 1924, a mass movement to demand
that lower caste persons be given the right to use a public path in front of the famous Vaikom temple.
 In the 1940s, he launched Dravidar Kazhagam, which espoused an independent Dravida Nadu comprising Tamil,
Malayalam, Telugu, and Kannada speakers.
 As a social reformer, he focused on social, cultural and gender inequalities, and his reform agenda questioned
matters of faith, gender and tradition.
 He asked people to be rational in their life choices. He argued that women needed to be independent, not mere
child-bearers, and insisted that they be allowed an equal share in employment.
Hence, option A is correct.

Q9. Consider the following statements with reference to the Lok Sabha:
1. The Speaker and the Deputy Speaker are the Presiding Officers of the Lok Sabha.
2. The Deputy Speaker presides over the Lok Sabha when the Speaker is absent from the sitting of the House.
3. In the ‘absence’ of the Speaker and the Deputy Speaker, the duty of the office of the Speaker is performed by such
member of the Lok Sabha as appointed by the President.
4. When the offices of both the Speaker and the Deputy Speaker fall ‘vacant’, the House shall be presided by a Chairperson
from amongst the panel nominated by the Speaker of the Lok Sabha.
Which of the statements given above are correct?

2nd Floor, 45 Pusa Road, Opp. Metro Pillar 128, Karol Bagh, New Delhi-110005
Ph: 08045248491, 7041021151 | Email: students@levelupias.com
A. 1, 2 and 3 only
B. 3 and 4 only
C. 1 and 2 only
D. 1, 2, 3 and 4
Answer: C
Explanation:
 Statement 1 is correct: The Speaker and the Deputy Speaker are the two primary presiding officers of the Lok
Sabha. The Speaker, elected by the members of the Lok Sabha, is the principal presiding officer and is responsible
for maintaining order in the house, deciding who may speak, and ensuring that the rules of the house are followed.
The Deputy Speaker, also elected by the members, assists the Speaker in the conduct of the proceedings of the
house.
 Statement 2 is correct: When the Speaker is absent from a sitting of the Lok Sabha, the Deputy Speaker presides
over the proceedings. This ensures that there is always a presiding officer present to conduct the business of the
house.
 Statement 3 is not correct: In the absence of both the Speaker and the Deputy Speaker, the House is presided
over by a chairperson from amongst the panel nominated by the Speaker of the Lok Sabha. This is in accordance
with Article 95 of the Indian Constitution.
 Statement 4 is not correct: According to Article 95 of the Indian Constitution, when the offices of both the
Speaker and the Deputy Speaker fall vacant, the duties of the Office of the Speaker are performed by such
member of the Lok Sabha as the President may appoint for the purpose.
Hence, option C is correct.

Q10. Consider the following statements:


1. Political equality involves granting equal citizenship to all the members of the state.
2. Social equality involves guaranteeing certain minimum conditions of life to all the members of the society.
3. Economic equality involves guaranteeing absolute equality of wealth or income to all the members of the state.
How many of the above statements is/are correct?
A. Only one
B. Only two
C. All three
D. None
Answer: B
Explanation:
 Statement 1 is correct: Political equality refers to the idea that all citizens should have equal political rights and
opportunities. This includes the right to vote, freedom of expression, movement, association, and belief. In
democratic societies, political equality is typically enshrined in laws and constitutions to ensure that all citizens
have an equal voice in government and the ability to participate in the political process.

2nd Floor, 45 Pusa Road, Opp. Metro Pillar 128, Karol Bagh, New Delhi-110005
Ph: 08045248491, 7041021151 | Email: students@levelupias.com
 Statement 2 is correct: Social equality involve guaranteeing certain minimum conditions of life, such as
adequate healthcare, education, nourishment, and a minimum wage, to all members of society. It also involves
minimizing the effects of social and economic inequalities to ensure that all individuals have a fair and equal
chance to compete for opportunities. Social equality aims to create a level playing field where individuals can
pursue their goals and aspirations without facing discrimination or barriers based on their social background.
 Statement 3 is not correct: Economic equality does not aim to guarantee absolute equality of wealth or income.
Instead, it seeks to address significant differences in wealth, property, or income between individuals or classes
in society. Economic equality is about ensuring that all individuals have equal opportunities to improve their
economic condition and participate in the economy. This does not mean that everyone will have the same level
of wealth or income, but rather that everyone should have a fair chance to succeed based on their talents and
efforts.
Hence, option B is correct.

Q11. Consider the following statements:


1. The loans made by the Central Government to the State Governments are charged on the Consolidated Fund of India.
2. The pensions of the judges of the High Courts are charged on the Consolidated Fund of India.
3. The sums required to satisfy any judgement of any court is charged on the Consolidated Fund of India.
4. The expenditures which are charged on the Consolidated Fund of India can neither be voted upon nor discussed by the
Parliament.
Which of the statements given above is/are correct?
A. 1, 3 and 4 only
B. 2 and 3 only
C. 1, 2 and 4 only
D. 1, 2 and 3 only
Answer:
Explanation: D
 Statement 1 is correct: As per Article 293, loans made by the Central Government to State Governments are
considered expenditure of the central government and are charged on the Consolidated Fund of India.
 Statement 2 is correct: Pensions of High Court judges are a mandated expense of the government and are
charged on the Consolidated Fund of India, ensuring their financial security post-retirement.
 Statement 3 is correct: Any sums required to satisfy judgments, decrees, or awards of any court or arbitral
tribunal are charged on the Consolidated Fund of India. This ensures that the government has the necessary
funds to comply with legal obligations arising from court judgments, thereby upholding the rule of law.
 Statement 4 is not correct: While the expenditures which are charged on the Consolidated Fund of India are not
subjected to a vote, discussions on them are allowed in either House of Parliament. This ensures transparency
and parliamentary oversight despite the specific expenditures being predetermined. However, Parliament's
approval is not required for these mandated expenses. Parliament can, however, discuss how the government
utilizes these funds and potentially influence future policies.

2nd Floor, 45 Pusa Road, Opp. Metro Pillar 128, Karol Bagh, New Delhi-110005
Ph: 08045248491, 7041021151 | Email: students@levelupias.com
Hence, option D is correct.

Q12. Consider the following statements:


1. The Ordinance-making powers of the President go beyond the legislative power of the Parliament and accordingly, the
Ordinances can be issued on any entry under the Seventh Schedule or even beyond.
2. The Ordinance-making power of the Governor is co-extensive with the legislative power of the State Legislature and
accordingly, the Ordinances can be issued only on those subjects on which the State Legislature can make laws.
3. The Ordinance-making power of the President is not a discretionary power, but the Ordinance-making power of the
Governor is a discretionary power.
How many of the above statements is/are correct?
A. Only one
B. Only two
C. All three
D. None
Answer: A
Explanation:
 Statement 1 is not correct: The Ordinance-making power of the President, as per Article 123 of the Indian
Constitution, is co-extensive with the legislative power of Parliament. This means that the President can issue
ordinances only on those subjects on which Parliament can make laws. These subjects are specified in the Union
List and the Concurrent List of the Seventh Schedule of the Constitution.
 Statement 2 is correct: The Ordinance-making power of the Governor, under Article 213 of the Indian
Constitution, is co-extensive with the legislative power of the State Legislature. This means that the Governor
can issue ordinances only on those subjects on which the State Legislature can make laws. These subjects are
specified in the State List and the Concurrent List of the Seventh Schedule of the Constitution.
 Statement 3 is not correct: Both the President and the Governor can only promulgate or withdraw an Ordinance
on the advice of their Council of Ministers headed by the Prime Minister and the Chief Ministers, respectively.
Therefore, the Ordinance-making power of both the President and the Governor is not a discretionary power
but is exercised on the advice of their respective Council of Ministers.
Hence, option A is correct.

Q13. Consider the following statements with reference to the transfer of the judges of the High Court:
1. There can be a judicial review of the decision to transfer a judge from one High Court to another.
2. The Supreme Court Collegium for the transfer of the judges is the same as that for the appointment of the judges to
the High Court.
3. The transfer of the judges is an exceptional measure for punishing the judges with doubtful integrity.
How many of the above statements is/are correct?
A. Only one
B. Only two

2nd Floor, 45 Pusa Road, Opp. Metro Pillar 128, Karol Bagh, New Delhi-110005
Ph: 08045248491, 7041021151 | Email: students@levelupias.com
C. All three
D. None
Answer: A
Explanation:
The President can transfer a judge from one High Court to another, after consulting the Chief Justice of India. On
transfer, he is entitled to receive, in addition to his salary, such compensatory allowance as may be determined by the
Parliament.
 Statement 1 is correct: In 1994, the Supreme Court held that judicial review is necessary to check arbitrariness
in the transfer of judges. However, only the judge who is transferred can challenge it.
 Statement 2 is not correct: In the Third Judges case of 1998, the Supreme Court opined that for the transfer of
High Court judges, the Chief Justice of India should consult not only the Collegium of four senior-most judges of
the Supreme Court but also the Chief Justices of the two High Courts involved (the one from which the judge is
being transferred and the one receiving him).
 Statement 3 is not correct: In 1977, the Supreme Court ruled that the transfer of High Court judges could be
resorted to only as an exceptional measure and only in public interest, and not by way of punishment.
Hence, option A is correct.

Q14. Consider the following statements:


Statement-I:
The Constitution requires that a person must prove his majority in the legislative assembly before he is appointed as the
Chief Minister.
Statement-II:
A person who is not a member of the state legislature can be appointed as Chief Minister for six months.
Which one of the following is correct in respect of the above statements?
A. Both Statement-I and Statement-II are correct and Statement-II is the correct explanation for Statement-I.
B. Both Statement-I and Statement-II are correct and Statement-II is not the correct explanation for Statement-I.
C. Statement-I is correct but Statement-II is incorrect.
D. Statement-I is incorrect but Statement-II is correct.
Answer: D
Explanation:
 Statement-I is not correct: The Constitution does not explicitly require a person to prove their majority in the
legislative assembly before being appointed as the Chief Minister. The governor may first appoint him as the
Chief Minister and then ask him to prove his majority in the legislative assembly within a reasonable period.
 Statement-II is correct: A person who is not a member of the state legislature can be appointed as Chief Minister
for six months, within which time, he should be elected to the state legislature, failing which he ceases to be the
Chief Minister.
Hence, option D is correct.

2nd Floor, 45 Pusa Road, Opp. Metro Pillar 128, Karol Bagh, New Delhi-110005
Ph: 08045248491, 7041021151 | Email: students@levelupias.com
Q15. Which of the following statements is/are correct regarding the proposal containing the alternation of the name of
the state?
1. The proposal containing the alternation of the name of the state has to refer by the President to the Legislature of the
concerned state.
2. The State Legislature has to express its opinion within a period specified by the President.
3. The views expressed by the state do not bind the decisions of either the President or the Parliament.
Select the correct answer using the code given below:
A. 1 only
B. 2 only
C. 2 and 3 only
D. 1, 2 and 3
Answer: D
Explanation:
 Statement 1 is correct: As per Article 3 of the Indian Constitution, any proposal to change the name of a state
must be sent by the President to the legislature of the concerned state for expressing its views. This step ensures
that the state government, representing the interests of the people of that state, has an opportunity to review
and provide input on the proposed name change.
 Statement 2 is correct: The President sets a specified time for the state legislature to convey its views on the
proposal. This ensures that the process of considering the name change is timely and allows for efficient decision-
making.
 Statement 3 is correct: While the state's opinion is important, it is not binding on the decisions made by either
the President or the Parliament. Ultimately, the decision to approve or reject the proposed name change rests
with the central government (represented by the President) and the Parliament. The views of the state legislature
are part of the consultation process but do not determine the final outcome.
Hence, option D is correct.

Q16. Which of the following are NOT the consequences of the declaration of the President's Rule?
1. The powers of the State Legislature are exercised by the Parliament.
2. The State Legislature must be dissolved.
3. The law made by the Parliament continues to be operative only until the President's Rule is in operation.
4. There is no effect on the enjoyment of the Fundamental Rights.
Select the correct answer using the code given below:
A. 2 and 3 only
B. 3 and 4 only
C. 1, 3 and 4 only
D. 1, 2 and 4 only
Answer: A

2nd Floor, 45 Pusa Road, Opp. Metro Pillar 128, Karol Bagh, New Delhi-110005
Ph: 08045248491, 7041021151 | Email: students@levelupias.com
Explanation:
 Statement 1 is correct: When the President's Rule is imposed in a state under Article 356 of the Constitution, the
state government and its legislature are suspended and the powers of the State Legislature are temporarily
transferred to the Parliament. This means that the Parliament can make laws on subjects that fall within the
jurisdiction of the State Legislature. This is necessary to ensure governance in the state when the normal
functioning of the state government is disrupted.
 Statement 2 is not correct: While it is true that the State Legislature can be dissolved during the President's Rule,
it is not a necessary consequence. The decision to dissolve or suspend the State Legislature depends on the
circumstances. Hence, the State Legislature can be dissolved or put under animated suspension. The House can
be revived also if a situation arises.
 Statement 3 is not correct: Laws made by the Parliament during the President's Rule remain valid even after the
President's Rule has been lifted. This ensures continuity and stability in governance.
 Statement 4 is correct: Unlike during a National Emergency, where Fundamental Rights can be suspended or
curtailed, there is no such effect on Fundamental Rights during the President's Rule. Citizens continue to enjoy
their Fundamental Rights as guaranteed by the Constitution.
Hence, option A is correct.

Q17. Consider the following statements with reference to Jet Streams:


1. These are narrow belt of high-altitude easterly winds in the troposphere.
2. They occur only in southern hemispheres due to presence of vast oceans.
3. They are the strongest during summer months in both the hemispheres.
4. The meandering jet streams are called as Rossby waves.
How many of the above statements is/are correct?
A. Only one
B. Only two
C. Only three
D. All four
Answer: A
Explanation:
 Statement 1 is not correct: Jet streams are
narrow belts of high-altitude westerly (not
easterly) winds in the troposphere, typically
around 7-12 miles above the Earth's surface.
They impact weather, air travel and many
other things that take place in our
atmosphere. Their speed varies from about
110 km/h in summer to about 184 km/h in
winter.

2nd Floor, 45 Pusa Road, Opp. Metro Pillar 128, Karol Bagh, New Delhi-110005
Ph: 08045248491, 7041021151 | Email: students@levelupias.com
 Statement 2 is not correct: Jet streams exist in both the northern and southern hemispheres.
 Statement 3 is not correct: Jet streams are strongest during winter months (not summers) in both hemispheres.
This is because the temperature difference between warm and cold air masses is greatest during winter,
creating stronger wind currents.
 Statement 4 is correct: The meandering jet streams are called as Rossby waves. These are natural phenomenon
in the atmosphere and oceans due to rotation of earth.
Hence, option A is correct.

Q18. “This climate has adequate rainfall throughout the year and tends towards a mild winter or autumn maximum from
cyclonic sources. The mean annual temperature is usually between 5°C and 15°C. The natural vegetation of this climatic
type is a deciduous forest. This region differs from many others in its unprecedented industrial advancement.”
Which of the following climatic type is described in the passage given above?
A. British Type Climate
B. Steppe Climate
C. Natal Climate
D. Mediterranean Climate
Answer: A
Explanation:
British Type Climate:
 Adequate rainfall throughout the year: The British Type Climate is characterized by relatively high levels of
precipitation throughout the year. This is due to the influence of prevailing westerly winds that bring moist air
from the Atlantic Ocean to regions like the British Isles. This consistent rainfall supports lush vegetation and
contributes to the formation of deciduous forests.
 Mild winters or autumn maximum from cyclonic sources: In this climate type, winters tend to be mild due to the
moderating influence of the ocean. Additionally, autumn maximum, which refers to a peak in precipitation
during the autumn months. This increase in rainfall is often associated with cyclonic activity, which is common
in maritime climates where low-pressure systems can develop over the ocean and move inland, bringing
precipitation.
 Mean annual temperature between 5°C and 15°C: The British Type Climate typically experiences moderate
temperatures throughout the year, with mean annual temperatures falling within the range between 5°C and
15°C. The influence of the ocean helps to regulate temperatures, leading to relatively mild winters and cooler
summers compared to continental climates.
 Natural vegetation of deciduous forest: Deciduous forests are characteristic of regions with temperate climates
like the British Type Climate. These forests are composed of trees that shed their leaves seasonally, typically in
response to changes in temperature and daylight. The consistent rainfall and moderate temperatures of the
maritime climate support the growth of deciduous trees.

2nd Floor, 45 Pusa Road, Opp. Metro Pillar 128, Karol Bagh, New Delhi-110005
Ph: 08045248491, 7041021151 | Email: students@levelupias.com
 Unprecedented industrial advancement: This phrase indicates that the region described has experienced
significant industrialization. The British Isles, which are prime examples of areas with a British Type Climate,
have a long history of industrial development, especially during the Industrial Revolution. The temperate
climate and abundant natural resources of these regions facilitated the growth of industries such as
manufacturing and textiles.

Hence, option A is correct.

Q19. Consider the following factors:


1. The Indian Ocean is bounded to the north by the Asian continent.
2. The Asian continent drives the strongest monsoon on the Earth.
3. The Indian Ocean lacks steady Equatorial easterlies.
How many of the above factors make the ocean circulation in the Indian Ocean different from the Atlantic and the Pacific
circulations?
A. Only one
B. Only two
C. All three
D. None
Answer: C
Explanation:

2nd Floor, 45 Pusa Road, Opp. Metro Pillar 128, Karol Bagh, New Delhi-110005
Ph: 08045248491, 7041021151 | Email: students@levelupias.com
 Statement 1 is correct: The presence of the Asian continent to the north of the Indian Ocean significantly impacts
its circulation. Unlike the Atlantic and Pacific Oceans, which have open connections to higher latitudes, the
Indian Ocean is more isolated. This isolation limits the northward heat export from the Indian Ocean and also
restricts the ventilation of the Indian Ocean thermocline (the layer of the ocean where temperature changes
rapidly with depth) from the north. This boundary condition influences the dynamics of the Indian Ocean, making
it unique compared to the other oceans.
 Statement 2 is correct: The Asian continent drives the strongest monsoon on Earth, characterized by seasonal
winds that reverse direction. These monsoonal winds have a profound effect on the Indian Ocean currents. For
example, during the Southwest Monsoon (summer), winds blow from the southwest, creating strong currents
that impact the ocean circulation. These currents, such as the Southwest Monsoon Current and the Northeast
Monsoon Current, are unique to the Indian Ocean and contribute to its distinctiveness.
 Statement 3 is correct: Unlike the Atlantic and Pacific Oceans, the Indian Ocean lacks steady Equatorial
easterlies. This is due to the rising branch of the Indian Ocean Walker circulation being anchored over the
maritime continent (Southeast Asia and nearby islands). The Walker circulation is a large-scale atmospheric
circulation pattern that influences the direction of winds and the distribution of precipitation around the Equator.
In the Indian Ocean, this circulation pattern results in the absence of consistent easterly winds along the Equator,
which is a notable difference from the other tropical oceans.
Hence, option C is correct.

Q20. Consider the following statements:


Statement-I:
Igneous rocks are also known as primary rocks.
Statement-II:
Fossil deposits are only found in igneous rocks.
Which one of the following is correct in respect of the above statements?
A. Both Statement-I and Statement-II are correct and Statement-II is the correct explanation for Statement-I.
B. Both Statement-I and Statement-II are correct and Statement-II is not the correct explanation for Statement-I.
C. Statement-I is correct but Statement-II is incorrect.
D. Statement-I is incorrect but Statement-II is correct.
Answer: C
Explanation:
 Statement-I is correct: Igneous rocks are formed from the solidification of molten material, either magma
beneath the Earth's surface or lava at the surface. This process of solidification is called crystallization. Because
igneous rocks form directly from molten material, they are considered primary rocks in the rock cycle. They are
the first rocks to form, making them primary in sequence.
 Statement-II is not correct: Fossils are the preserved remains of ancient organisms. Igneous rocks are formed
from molten material and wouldn't contain fossils. Fossil deposits are primarily found in sedimentary rocks.
Sedimentary rocks form from the accumulation and lithification (compaction and cementation) of sediments,
including particles of sand, silt, and clay, as well as organic remains like shells, bones, and plant material. Over

2nd Floor, 45 Pusa Road, Opp. Metro Pillar 128, Karol Bagh, New Delhi-110005
Ph: 08045248491, 7041021151 | Email: students@levelupias.com
time, these sediments can accumulate and become cemented together to form sedimentary rocks. Fossils are
preserved within these sedimentary layers, providing valuable information about past life forms and
environments.
o While it is theoretically possible for fossils to become encased in igneous rocks through various geological
processes (e.g., volcanic ash burial), this is far less common compared to their occurrence in sedimentary
rocks.
Hence, option C is correct.

Q21. You and your friend have started walking along the River Kaveri from Karnataka, where it originates, to its mouth at
the Indian Ocean. In this context, consider the following statements:
1. You will not come across any river islands on your way.
2. You will come across Pancharangams or five sacred Hindu temples on the banks on the river.
3. You will find Hogenakkal Falls at the border between Tamil Nadu and Karnataka.
Which of the statements given above is/are correct?
A. 1 Only
B. 1 and 2 Only
C. 1 and 3 Only
D. 2 and 3 Only
Answer: D
Explanation:
 Statement 1 is not correct: Kaveri River splits into two places in Karnataka and forms two islands. One is an
Shivanasamudra, where it plunges as Gagana Chukki and Bara Chukki Falls. Another is the island of Srirangapatna
near Mysore. The third island is Srirangam in Tamil Nadu.
 Statement 2 is correct: Pancharanga Kshetrams or Pancharangams is a group of five sacred Hindu temples on
the banks of river Kaveri River, dedicated to Ranganatha, a form of the God Vishnu. The Srirangapatna also
called Adi Rangam is the first temple on the banks of the Kaveri River from the upstream side. It is followed by
the Srirangam in Tamil Nadu.
 Statement 3 is correct: Hogenakal is situated at the borders of Karnataka and Tamil Nadu. Kaveri enters Tamil
Nadu as a big river with gushing water presentably as a natural fall. The name Hogenakal is derived from Kannada
and means ‘Smoky Rocks’ as the waterfall resembles smoke emanating from the rocks.
Hence, option D is correct.

2nd Floor, 45 Pusa Road, Opp. Metro Pillar 128, Karol Bagh, New Delhi-110005
Ph: 08045248491, 7041021151 | Email: students@levelupias.com
Q22. Consider the following pairs:

Valleys in news Related States

1. Dibang Valley Arunachal Pradesh

2. Araku Valley Meghalaya

3. Cumbum Valley Kerala

4. Barak Valley Assam

How many of the above pairs is/are correctly matched?


A. Only one
B. Only two
C. Only three
D. All four
Answer: B
Explanation:
 Pair 1 is correct: Dibang Valley is located in Arunachal Pradesh. It is known for its scenic beauty, rich biodiversity,
and as a part of the Eastern Himalayan region. It is home to the Dibang Wildlife Sanctuary.
 Pair 2 is not correct: Araku Valley is located in Andhra Pradesh. It is a famous hill station known for its picturesque
landscapes, coffee plantations, and indigenous tribal culture.
 Pair 3 is not correct: Cumbum Valley is located in Tamil Nadu (not Arunachal Pradesh). It is renowned for its
agriculture, particularly grape cultivation.
 Pair 4 is correct: Barak Valley is located in Assam. It is known for its cultural diversity and as an important
agricultural region in Assam. It is formed by the Barak River and is home to various ethnic communities.
Hence, option B is correct.

Q23. Which of the following is/are said to increase the Current Account Deficit (CAD) of India?
1. A foreign national visiting a yoga camp in Haridwar.
2. An Indian national going to study abroad.
3. Rise in international oil prices.
Select the correct answer using the code given below:
A. 1, 2 and 3
B. 1 and 3 only
C. 1 only
D. 2 and 3 only
Answer: D
Explanation:

2nd Floor, 45 Pusa Road, Opp. Metro Pillar 128, Karol Bagh, New Delhi-110005
Ph: 08045248491, 7041021151 | Email: students@levelupias.com
 Statement 1 is not correct: When a foreign national visits India for a yoga camp, they bring foreign currency into
the country. This scenario represents an export of services from India. This foreign currency is typically spent on
various goods and services during their stay, including accommodation, food, transportation, and other expenses
related to the yoga camp. These expenditures contribute to the inflow of foreign currency into the Indian
economy. Since this scenario involves the inflow of foreign currency, it does not contribute to increasing the
current account deficit. Instead, it helps to improve the current account balance by boosting foreign exchange
reserves.
 Statement 2 is correct: When an Indian national goes abroad for higher education, they typically incur expenses
in the currency of the host country, such as dollars, euros, or pounds. This scenario represents an import of
services for India. The expenses include tuition fees, accommodation, living expenses, and other costs associated
with studying abroad. As a result, there is an outflow of foreign currency from India to the host country. This
outflow of foreign currency contributes to increasing the current account deficit because it represents a negative
balance in the current account due to higher expenditures compared to receipts from abroad.
 Statement 3 is correct: India is a major importer of crude oil and petroleum products. When there is a rise in
international oil prices, India needs to spend more foreign currency to purchase oil from abroad. Since oil is a
crucial commodity for various sectors of the economy, including transportation, manufacturing, and agriculture,
the increase in oil prices leads to higher import costs for India. As a result, there is a higher outflow of foreign
currency from India to oil-exporting countries, contributing to increasing the current account deficit.
Hence, option D is correct.

Q24. Consider the following statements with reference to the Retail Direct Scheme of RBI:
1. Individual Retail investors can open ‘Retail Direct Gilt Account’ (RDG Account) with the RBI.
2. Non-resident retail investors are not eligible.
3. The RBI has imposed minimum and maximum investment limits for the retail investors.
4. A retail investor can take a loan against the securities available in the RDG account.
How many of the above statements is/are correct?
A. Only one
B. Only two
C. Only three
D. All four
Answer: C
Explanation:
 Statement 1 is correct: The RBI's Retail Direct Scheme allows individual retail investors to open a Gilt Securities
Account, also known as the Retail Direct Gilt (RDG) account, directly with the Reserve Bank of India (RBI).
Through this account, retail investors can participate in buying Government Securities (G-Secs) such as Treasury
Bills (T-Bills), Dated Securities (D-Secs), State Development Loans (SDLs), and Sovereign Gold Bonds.
 Statement 2 is not correct: Non-Resident retail investors are also eligible to participate in the Retail Direct
Scheme by opening an RDG account with the RBI. However, they must comply with the necessary Know Your
Customer (KYC) norms and other requirements set forth by the RBI.

2nd Floor, 45 Pusa Road, Opp. Metro Pillar 128, Karol Bagh, New Delhi-110005
Ph: 08045248491, 7041021151 | Email: students@levelupias.com
 Statement 3 is correct: The RBI has imposed minimum and maximum investment limits for retail investors
participating in the Retail Direct Scheme.
o For example, there is a minimum investment requirement of Rs 10,000 for G-Secs. While the exact limits
may vary depending on the specific type of security and other factors, the RBI establishes these limits to
regulate and manage retail investors' participation in the government securities market.
 Statement 4 is correct: Retail investors can avail of loans against securities in their RDG accounts. Retail investors
can gift government securities to other retail investors and nominate up to two individuals to receive their
securities in case of an unfortunate event.
Hence, option C is correct.

Q25. Consider the following statements with reference to Foreign Direct Investment (FDI) and Foreign Institutional
Investment (FII):
1. While FDI’s are short-term investments, the FII’s are long term investments.
2. FII grants investors substantial management control over businesses in the foreign country.
3. In comparison to FDI, FIIs can quickly enter and exit the stock market.
How many of the above statements is/are correct?
A. Only one
B. Only two
C. All three
D. None
Answer: A
Explanation:
FDI and FII are both forms of foreign investment, but they differ significantly in their operations, target audience, and
potential returns. FDI is defined as the investment made by a company if the company is situated outside the country. FII
is when investors, most commonly in the form of institutions that invest in the country’s financial market.

 Statement 1 is not correct: FDI are typically long-term investments, often involving establishing physical
operations or acquiring significant stakes in companies. FII can be both short-term and long-term, depending on
the investor's strategy.
 Statement 2 is not correct: It is FDI (not FII) which grants investors substantial management control over
businesses in the foreign country. FDI being a direct form of investment into a foreign company, the investors
are more interested and even get to enjoy a higher control on the management of the company, even if it is in
the foreign country.

 Statement 3 is correct: FII investments can be readily bought and sold on stock exchanges, facilitating quicker
entry and exit compared to FDI. Establishing and divesting physical operations involved in FDI often takes more
time and effort.
Hence, option A is correct.

2nd Floor, 45 Pusa Road, Opp. Metro Pillar 128, Karol Bagh, New Delhi-110005
Ph: 08045248491, 7041021151 | Email: students@levelupias.com
Q26. Consider the following statements with reference to the Balance of Payments (BoP):
1. The current account measures the transfer of real resources (goods, services, income and transfers) between an
economy and the rest of the world.
2. The capital and financial account reflects the net changes in financial claims on the rest of the world.
3. The compilation and dissemination of BoP data is the prime responsibility of the Reserve Bank of India (RBI).
How many of the above statements is/are correct?
A. Only one
B. Only two
C. All three
D. None
Answer: C
Explanation:
Balance of Payments (BoP) statistics systematically summarise, for a specific period, the economic transactions of an
economy with the rest of the world. The BoP can be broadly divided into two accounts - (1) Current account and (2)
Capital and Financial account.
 Statement 1 is correct: The current account measures the transfer of real resources (goods, services, income and
transfers) between an economy and the rest of the world.
o The current account is further subdivided into merchandise account and invisibles account. The
merchandise account consists of transactions relating to exports and imports of goods. In the invisible
account, there are three broad categories, namely
 Non-factor services, such as travel, transportation, insurance and miscellaneous services;
 Transfers which do not involve any value in exchange; and
 Income which includes compensation of employees and investment income.
 Statement 2 is correct: The capital and financial account reflects the net changes in financial claims on the rest of
the world.
o The capital account can be broadly broken up into two categories,
 Non-debt flows, such as direct and portfolio investments; and
 Debt flows, such as external assistance, commercial borrowings, non-resident deposits, etc. The sum
of the current account and capital account indicates the overall balance, which could either be in
surplus or in deficit. The movement in overall balance is reflected in changes in the international
reserves of the country.
 Statement 3 is correct: The compilation and dissemination of BoP data is the prime responsibility of RBI.
Hence, option C is correct.

2nd Floor, 45 Pusa Road, Opp. Metro Pillar 128, Karol Bagh, New Delhi-110005
Ph: 08045248491, 7041021151 | Email: students@levelupias.com
Q27. Consider the following statements with reference to Bond Yield Inversion :
1. It occurs when yield on a longer tenure bond becomes less than the yield for a shorter tenure bond.
2. An inverted yield curve is a strong indicator of an impending recession.
3. Bond Yield is inversely proportional to the Bond Price.
How many of the above statements is/are NOT correct?
A. Only one
B. Only two
C. All three
D. None
Answer: D
Explanation:
 Statement 1 is correct: Typically, long-term bonds have higher yields than short-term bonds, and the yield curve
slopes upward to the right. An inverted yield curve represents a situation in which long-term debt instruments
have lower yields than short-term debt instruments of the same credit quality. When the curve inverts, it signifies
a shift in expectations, with investors demanding higher returns for short-term investments compared to long-
term ones.
 Statement 2 is correct: An inverted yield curve is considered a strong indicator of an impending recession. This
is because it suggests that investors expect interest rates to fall in the future, which could be due to expectations
of weaker economic growth.
 Statement 3 is correct: Bond yield and bond price have an inverse relationship. When bond yields rise, bond
prices fall, and vice versa. This is because bond yields represent the return investors receive on their investment,
so when yields go up, the existing bonds with lower yields become less attractive, leading to a decrease in their
prices.
Hence, option D is correct.

Q28. Consider the following statements with reference to the possible benefits of internationalisation of the Indian rupee:
1. Various central banks may start accepting the rupee as a reserve currency.
2. The rupee will be usable in the repayment of bonds issued globally.
3. India will be able to use more rupee for the payment of its imports.
How many of the above statements is/are correct?
A. Only one
B. Only two
C. All three
D. None
Answer: C
Explanation:

2nd Floor, 45 Pusa Road, Opp. Metro Pillar 128, Karol Bagh, New Delhi-110005
Ph: 08045248491, 7041021151 | Email: students@levelupias.com
 Statement 1 is correct: A reserve currency is a foreign currency that central banks hold as part of their reserves.
If the rupee becomes more widely used in international trade, it becomes more attractive for central banks to
hold rupees as part of their reserves. This increases demand for the rupee and can strengthen its value.
 Statement 2 is correct: When bonds are issued internationally in a particular currency, the issuer (in this case,
India) commits to repaying the principal amount and interest in that currency. If the rupee is internationalized
and used for issuing bonds, it would mean that India could repay these bonds using its own currency. This
reduces the exchange rate risk for the Indian government or corporates issuing these bonds, as they don't have
to worry about fluctuations in the exchange rate affecting the repayment amount. This can make borrowing in
international markets more attractive for Indian entities.
 Statement 3 is correct: Currently, most international trade transactions are conducted in major currencies like
the US dollar, euro, or Japanese yen. If the Indian rupee becomes more widely accepted for international trade,
it would mean that Indian businesses can invoice and settle payments in rupees. This simplifies transactions
and reduces the costs associated with currency conversion and hedging against exchange rate fluctuations. It
also promotes the use of the rupee in global trade and finance, enhancing India's economic influence on the global
stage.
Hence, option C is correct.

Q29. Consider the following statements:


Statement-I:
Elimination of Revenue deficit is good for Indian Economy as it will ensure that all the borrowings are for capital
expenditure.
Statement-II:
For India, Revenue expenditure has a fiscal multiplier of less than 1, whereas capital expenditure has a fiscal multiplier of
more than 2.
Which one of the following is correct in respect of the above statements?
A. Both Statement-I and Statement-II are correct and Statement-II is the correct explanation for Statement-I.
B. Both Statement-I and Statement-II are correct and Statement-II is not the correct explanation for Statement-I.
C. Statement-I is correct but Statement-II is incorrect.
D. Statement-I is incorrect but Statement-II is correct.
Answer: A
Explanation:
 Statement-I is correct: Eliminating the revenue deficit is generally good for the economy. A revenue deficit
means the government is spending more than it collects in taxes, leading to borrowing to cover current expenses.
This can crowd out private investment and lead to higher interest rates.
o Eliminating the deficit frees up funds for capital expenditure, which can be positive for long-term
growth. Capital expenditure refers to government spending on infrastructure, education, and other
investments that create a return over time.
 Statement-II is correct: A paper, titled “Fiscal Multipliers for India” by Sukanya Bose and N R Bhanumurthy
shows, the multiplier is less than 1 for revenue expenditure and over 2.5 for capital expenditure.

2nd Floor, 45 Pusa Road, Opp. Metro Pillar 128, Karol Bagh, New Delhi-110005
Ph: 08045248491, 7041021151 | Email: students@levelupias.com
o In other words, when the government spends Rs 100 on increasing salaries in India, the economy grows
by a little less than Rs 100. But, when the government uses that money to make a road or a bridge, the
economy’s GDP grows by Rs 250
Hence, option A is correct.

Q30. Consider the following statements:


1. When the value of the currency is made cheaper by the central bank it is called depreciation of the currency, and when
the market forces bring down the value of the currency due to falling demand it is called devaluation of the currency.
2. In the Balance of Payments, the movements of money without exchange for goods or services and charities are part of
Capital account.
Which of the statements given above is/are correct?
A. 1 only
B. 2 only
C. Both 1 and 2
D. Neither 1 nor 2
Answer: D
Explanation:
 Statement 1 is not correct: Depreciation refers to a decline in the value of a currency in a floating exchange rate
system, where the exchange rate is determined by market forces of supply and demand. Depreciation occurs
naturally as a result of market dynamics. Factors such as changes in economic indicators (like inflation, interest
rates, or economic growth), shifts in trade balances, political stability, and market speculation can influence supply
and demand for a currency, leading to changes in its value relative to other currencies.
o Devaluation refers to a deliberate downward adjustment in the official value of a currency by a
government or central bank. It is implemented by central authorities to address specific economic
objectives, such as improving trade balances or stimulating economic growth. By reducing the value of
its currency, a country can make its exports cheaper and imports more expensive, which may help boost
domestic industries and reduce trade deficits.
 Statement 2 is not correct: In the Balance of Payments (BoP), the current account records transactions involving
goods, services, income, and unilateral transfers.
o Remittances and charitable donations fall under unilateral transfers, which are transactions where one
party provides something of value to another without receiving anything in return. These transactions are
considered part of the Current account because they involve the movement of money without the
exchange of goods or services.
Hence, option D is correct.

2nd Floor, 45 Pusa Road, Opp. Metro Pillar 128, Karol Bagh, New Delhi-110005
Ph: 08045248491, 7041021151 | Email: students@levelupias.com
Q31. Consider the following statements:
1. Under the Ramsar Convention, if a wetland supports 1% of the individuals in a population of one species or subspecies
of waterbird, it can be a criteria for designating the wetland as the wetland of international importance.
2. Chandra Taal is the smallest Ramsar site in India.
3. As of April 2024, Uttar Pradesh has the highest number of Ramsar sites in India.
How many of the above statements is/are correct?
A. Only one
B. Only two
C. All three
D. None
Answer: A
Explanation:
Ramsar Convention, also known as the Convention on Wetlands, is an intergovernmental environmental treaty
established in 1971 by UNESCO, which came into force in 1975. A Ramsar site is a wetland site of international importance
under the Ramsar Convention.
 Statement 1 is correct: The Ramsar Convention has specific criteria for designating wetlands as internationally
important Ramsar Sites. One criterion is based on waterbirds. A wetland qualifies if it regularly supports at least
1% of the global population of one waterbird species or subspecies.
o This criterion highlights the significance of wetlands for migratory birds and aims to protect critical
habitats for these populations.
 Statement 2 is not correct: Renuka Wetland is India's smallest wetland site. It covers an area of only 0.2 square
Kilometer. The lake is home to at least 443 species of fauna and 19 species of ichthyofauna representative of
lacustrine ecosystems, as well as a variety of flora.
o The wetland is also a sacred place for Hindus, and is named after the goddess Renuka, the wife of the
sage Jamadagni. According to legend, the lake was formed when sage Parshuram obeyed his father and
sacrificed his mother, Renuka. An annual fair is held at the lake in November to celebrate the immortality
of Renuka and her son.
 Statement 3 is not correct: As of 2024, India has 80 Ramsar site. Tamil Nadu currently holds the top spot with 16
Ramsar sites, followed by Uttar Pradesh with 10.
Hence, option A is correct.

Q32. Consider the following statements with respect to Gangetic River Dolphin:
1. It can live only in freshwater and is essentially blind.
2. It is recognized as National Aquatic Animal of India.
3. Its IUCN status is critically endangered as it is hunted for meat and oil.
4. Being a mammal, it cannot breathe in water and must come on surface frequently to breath.
How many of the above statements is/are NOT correct?
A. Only one

2nd Floor, 45 Pusa Road, Opp. Metro Pillar 128, Karol Bagh, New Delhi-110005
Ph: 08045248491, 7041021151 | Email: students@levelupias.com
B. Only two
C. Only three
D. All four
Answer: A
Explanation:
 Statement 1 is correct: The Gangetic dolphins can only live in freshwater and are essentially blind. They hunt by
emitting ultrasonic sounds waves that bounce off of fish and other prey. They are indicator of a healthy ecosystem.
 Statement 2 is correct: It is recognized as National Aquatic Animal of India. It is found in the river systems of
Ganga, Brahmaputra, Meghna, and Karnaphuli-Sangu in Nepal, India, and Bangladesh.
 Statement 3 is not correct: Its IUCN status is Endangered (not critically endangered), though it faces significant
threat from hunting, overfishing, construction of dams, dredging etc.
o India’s first dolphin research facility was inaugurated in Patna, Bihar.
 Statement 4 is correct: Being a mammal, it cannot breathe in water and must surface every 30-120 seconds. This
characteristic breathing pattern is one of the reasons the dolphin is nicknamed "Susu," which reflects the sound
it makes when coming up for air.
Hence, option A is correct.

Q33. Consider the following fauna:


1. Whales
2. Bats
3. Dolphins
4. Oilbirds
5. Swiftlets
How many of the above animals uses Echolocation?
A. Only two
B. Only three
C. Only four
D. All five
Answer: D
Explanation:
Echolocation is the technique of using sound signals to determine the location of objects through the echoes. It is also
known as biosonar. It is widely used for underwater navigation, communications, and even as a method to find fish. It
empowers animals to navigate seamlessly in absolute darkness, supporting activities like hunting, discerning friends from
foes, and evading obstacles.
 Whales, bats, dolphins, birds like oilbirds and swiftlets etc. are all known to echolocate.
 Oilbird is a nocturnal bird of South America, that lives in caves and feeds on fruits. It uses echolocation to find its
way within the caves.

2nd Floor, 45 Pusa Road, Opp. Metro Pillar 128, Karol Bagh, New Delhi-110005
Ph: 08045248491, 7041021151 | Email: students@levelupias.com
 Swiftlets are diurnal, insect-eating bird, use echolocation to locate their roost in dark caves.
Hence, option D is correct.

Q34. Consider the following statements:


Statement-I:
The energy pyramid of an ecosystem is always upright and narrows to the top.
Statement-II:
The biomass in the upper trophic levels is generally very high as compared to the lower trophic levels.
Which one of the following is correct in respect of the above statements?
A. Both Statement-I and Statement-II are correct and Statement-II is the correct explanation for Statement-I.
B. Both Statement-I and Statement-II are correct and Statement-II is not the correct explanation for Statement-I.
C. Statement-I is correct but Statement-II is incorrect.
D. Statement-I is incorrect but Statement-II is correct.
Answer: C
Explanation:
 Statement-I is correct: The energy pyramid represents the flow of energy through different trophic levels in an
ecosystem. At each trophic level, only a fraction of the energy from the previous level is passed on, as some
energy is lost as heat during metabolism and other processes. This results in the energy pyramid being upright,
with the energy decreasing as we move up the pyramid, and narrower at the top because less energy is available
to higher trophic levels.
 Statement-II is not correct: In most ecosystems (barring some aquatic ecosystems), the biomass (total mass of
organisms) decreases as we move up the trophic levels. This is because energy is lost at each transfer, so there
is less energy available to support organisms at higher trophic levels. As a result, the biomass of producers
(plants) is usually higher than the biomass of primary consumers (herbivores), and so on.
Hence, option C is correct.

Q35. "This national park is nestled between the Patkai and Dapha bum ranges of Mishmi Hills in the Eastern Himalayan
Sub-region. It is the world's only park with the four feline species of big cat, the Tiger, Leopard, Snow Leopard, and Clouded
Leopard, as well as a large number of Lesser cats. It is also home to a large number of the distinctive Hoolock Gibbons,
India's only "ape" species".
Identify this national park:
A. Manas National Park
B. Kaziranga National Park
C. Namdapha National Park
D. Raimona National Park
Answer: C
Explanation:

2nd Floor, 45 Pusa Road, Opp. Metro Pillar 128, Karol Bagh, New Delhi-110005
Ph: 08045248491, 7041021151 | Email: students@levelupias.com
Namdapha National Park:
 Namdapha National Park is located in the northeastern state of Arunachal Pradesh, India. It is nestled between
the Patkai and Dapha Bum ranges of the Mishmi Hills in the Eastern Himalayan Sub-region. The park is known
for its rich biodiversity and is one of the largest protected areas in the Eastern Himalaya biodiversity hotspot.
 One of the most remarkable features of Namdapha National Park is its diverse array of big cats. It is the only park
in the world that is home to four different species of big cats: the Tiger, Leopard, Snow Leopard, and Clouded
Leopard. These majestic animals find a suitable habitat in the rugged terrain and dense forests of the park.
 Apart from the big cats, it is also known for its population of Lesser cats, which include species like the Leopard
Cat and the Marbled Cat. The park is a haven for wildlife enthusiasts and researchers interested in studying these
elusive creatures.
 Another notable inhabitant of this park is the Hoolock Gibbon, India's only "ape" species. These agile primates
are found in good numbers in the park and are a delight to observe in their natural habitat.
 Namdapha National Park is not just about its charismatic megafauna; it is also home to a wide variety of flora
and fauna. The park boasts a rich diversity of plant species, including many rare and endemic ones. The varied
topography of the park, ranging from lowland tropical rainforests to alpine meadows, contributes to this rich
biodiversity.
 The park is also of great importance for conservation as it provides a vital habitat for many endangered species.
It is part of the Eastern Himalaya Endemic Bird Area and is home to several rare and endangered bird species,
making it a paradise for birdwatchers.
 In addition to its biological significance, Namdapha National Park also holds cultural importance. It is home to
several indigenous tribes, including the Lisus, Singphos, and Tangsas, who have been living in harmony with
nature for centuries.
Hence, option C is correct.

Q36. Through which of the following processes, energy can be extracted from municipal and industrial solid wastes?
1. Incineration
2. Gasification
3. Pyrolysis
4. Anaerobic digestion
Select the correct answer using the code given below:
A. 1 and 3 only
B. 1, 2 and 3 only
C. 2 and 4 only
D. 1, 2, 3 and 4
Answer: D
Explanation:
All of the listed processes can be used to extract energy from municipal and industrial solid wastes.

2nd Floor, 45 Pusa Road, Opp. Metro Pillar 128, Karol Bagh, New Delhi-110005
Ph: 08045248491, 7041021151 | Email: students@levelupias.com
 Incineration is a waste treatment process that involves the combustion of organic substances contained in waste
materials. Incineration of waste materials converts the waste into ash, flue gas and heat. The heat generated by
incineration can be used to generate electric power.
 Gasification also is known as Coal Gasification is the process where chemically transforms the fossil fuel into
Synthetic Natural Gas (SNG), instead of burning fossil fuel. It produces Syngas which is mixture consisting
primarily of methane (CH4), carbon monoxide (CO), hydrogen (H2), carbon dioxide (CO2) and water vapour (H2O).
 Pyrolysis is the thermal decomposition of materials at elevated temperatures in an inert atmosphere. It involves
a change of chemical composition. It is considered as the first step in the processes of gasification or combustion.
 Anaerobic digestion is a sequence of processes by which microorganisms break down biodegradable material in
the absence of oxygen. The process is used for industrial or domestic purposes to manage waste or to produce
fuels.
Hence, option D is correct.

Q37. Consider the following statements about the recently launched Green Credit Program (GCP):
1. The GCP will contribute to build a national market in which there is fungibility in green actions.
2. It is an initiative under Ministry of New and Renewable Energy.
3. The program takes ahead the LiFE (Lifestyle for Environment) movement.
4. The Indian Council of Forestry Research and Education serves as the administrator of the Scheme.
Which of the above statements is/are correct?
A. 1 only
B. 2 and 3 only
C. 1, 3 and 4 only
D. 3 and 4 only
Answer: C
Explanation:
 Statement 1 is correct: Fungibility refers to the ability of a good or asset to be readily interchanged for another
of like kind.
o Under GCP, the Green Credit earned through one mechanism (let’s say afforestation) can be used for
compensating soil degradation. So, there is fungibility in green actions.
 Statement 2 is not correct: It is an initiative of Ministry of Environment, Forest and Climate Change (MoEFCC).
 Statement 3 is correct: The program takes ahead the LiFE (Lifestyle for Environment) movement and incentivizes
environmental actions across diverse sectors by various stakeholder. It nudges individual and collective action
to protect and preserve the environment.
 Statement 4 is correct: The Indian Council for Forestry Research and Education (ICFRE) serves as the GCP
administrator, responsible for program implementation, management, monitoring and operation.
Hence, option C is correct.

2nd Floor, 45 Pusa Road, Opp. Metro Pillar 128, Karol Bagh, New Delhi-110005
Ph: 08045248491, 7041021151 | Email: students@levelupias.com
Q38. Consider the following statements with reference to the Bio-Medical Waste Management Rules, 2016:
1. The Rules cover all bio-medical wastes, including the medical instruments, but exclude the animals’ anatomical waste.
2. They promote the replacement of the conventional plastic bags with chlorinated plastic bags.
3. The Rules are under the ambit of the Environment (Protection) Act, 1986.
Which of the statements given above is/are correct?
A. 2 only
B. 1 and 3 only
C. 2 and 3 only
D. 3 only
Answer: D
Explanation:
 Statement 1 is not correct: The Bio-Medical Waste Management Rules, 2016, cover a wide range of bio-medical
wastes, including human and animal anatomical waste, treatment apparatus like needles, syringes, and other
materials used in healthcare facilities. This includes medical instruments used in diagnosis, treatment, or
immunization in hospitals, nursing homes, pathological laboratories, blood banks, and other healthcare
facilities.
 Statement 2 is not correct: The rules promote the phase-out of chlorinated plastic bags, gloves, and blood bags
within two years.
o Chlorinated plastic bags are harmful to the environment and can release toxic chemicals when
incinerated, contributing to pollution. The rules aim to replace these bags with safer alternatives to
minimize environmental impact.
 Statement 3 is correct: The Bio-Medical Waste Management Rules, 2016, are framed under the Environment
(Protection) Act, 1986. This Act provides the legal framework for environmental protection and conservation in
India. The rules aim to regulate the management and disposal of bio-medical waste to ensure the protection of
human health and the environment.
Hence, option D is correct.

Q39. Consider the following statements with reference to the Patenting Trends Report:
1. The report is released by World Intellectual Property Organization (WIPO).
2. As per the latest version of the report, no patents have been granted in India in the field of Deep technology like Artificial
Intelligence, Machine Learning etc.
Which of the above statements is/are correct?
A. 1 only
B. 2 only
C. Both 1 and 2
D. Neither 1 nor 2
Answer: D

2nd Floor, 45 Pusa Road, Opp. Metro Pillar 128, Karol Bagh, New Delhi-110005
Ph: 08045248491, 7041021151 | Email: students@levelupias.com
Explanation:
 Statement 1 is not correct: The report on Patenting Trends was released by NASSCOM (National Association of
Software and Service Companies). It is an Indian trade association that represents the IT and BPO industries in
India.
 Statement 2 is not correct: As per the report, India witnessed 83,000 patents being filed in FY2023, marking an
annual growth rate of 24.6%, the highest in the last two decades.
o The number of patents granted also witnessed significant growth rising over 2X between FY2019-
FY2023. This trend was expected to increase significantly with over 1,00,000 patents granted between
15th March 2023 and 14th March 2024.
o Among the top technology patents, Deep tech companies are filing patents for Artificial Intelligence,
the IoT and Neurotechnology.
Hence, option D is correct.

Q40. Consider the following statements with reference to Higgs Boson:


1. It is a subatomic particle that gives mass to other fundamental particles.
2. It has the longest lifespan among all the elementary particles.
3. It was discovered in 2012 at the Large Hadron Collider at CERN.
Which of the above statements is/are correct?
A. 1 and 2 only
B. 3 only
C. 1 and 3 only
D. 1, 2 and 3
Answer: C
Explanation:
 Statement 1 is correct: The Higgs Boson is a subatomic particle theorized to be responsible for giving mass to
other fundamental particles. It interacts with particles through the Higgs field, and this interaction is believed to
be the reason some particles have mass while others don't.
 Statement 2 is not correct: The Higgs Boson has very short lifespan. It is unstable and lives only for an extremely
short time, 1.6 x 10-22 seconds. It decays into other particles almost instantaneously after being created.
 Statement 3 is correct: The Higgs boson was proposed in 1964 by Peter Higgs, François Englert, and four other
theorists to explain why certain particles have mass. Scientists confirmed its existence in 2012 through the ATLAS
and CMS experiments at the Large Hadron Collider (LHC) at CERN in Switzerland. This discovery led to the 2013
Nobel Prize in Physics being awarded to Higgs and Englert.
Hence, option C is correct.

2nd Floor, 45 Pusa Road, Opp. Metro Pillar 128, Karol Bagh, New Delhi-110005
Ph: 08045248491, 7041021151 | Email: students@levelupias.com
Q41. Consider the following statements about Prostate:
1. Prostate gland is responsible for the production of sperms in men.
2. Only men have prostate and women don't have prostate gland.
3. Prostate cancer cases are on the downward trend globally in recent years.
How many of the above statements is/are correct?
A. Only one
B. Only two
C. All three
D. None
Answer: A
Explanation:
 Statement 1 is not correct: The prostate gland does not produce sperm. Sperm is produced in the testicles. The
prostate gland does, however, produce seminal fluid, which nourishes and protects sperm.
 Statement 2 is correct: Women don't have prostate gland. Women and people Assigned Female at Birth (AFAB)
have Skene's gland. However, some people refer to Skene's gland as the female prostate gland.
 Statement 3 is not correct: Prostate cancer cases are increasing as per a Lancet report in April 2024.
o Globally cases are expected to double from 1.4 million per year in 2020 to 2.9 million per year by 2040
with low- and middle-income countries predicted to see the highest increase
Hence, option A is correct.

Q42. Consider the following statements with reference to Cosmic Microwave Background Radiation:
1. It is the faint afterglow of the Big Bang.
2. It gives evidence of the expanding universe.
3. It is completely smooth and uniformly distributed throughout the universe.
Which of the above statements is/are correct?
A. 1 and 2 only
B. 1 and 3 only
C. 2 and 3 only
D. 1, 2 and 3
Answer: A
Explanation:
 Statement 1 is correct: Cosmic Microwave Background (CMB) is a cooled remnant of the first light that could
ever travel freely throughout the universe. This fossil radiation is the ‘furthest that any telescope can see’ and
was released soon after the ‘Big Bang’.
o CMBR is electromagnetic radiation as a remnant from an early stage of the Universe in Big Bang
Cosmology. It is an all-pervasive, but weak, electromagnetic radiation from early universe, when matter
was still to be formed.

2nd Floor, 45 Pusa Road, Opp. Metro Pillar 128, Karol Bagh, New Delhi-110005
Ph: 08045248491, 7041021151 | Email: students@levelupias.com
 Statement 2 is correct: CMB also gives evidence that Universe expanded from an initial violent explosion. Cosmic
Microwave radiation have become less energetic due to the redshift which also gives evidence of expanding
universe.
 Statement 3 is not correct: The radiation is mostly smooth (not completely). It contains some wiggles or
deformities, it its shape. These wiggles encode information about specific events that can be expected to be found
from in the CMB spectrum in different scenarios.
Hence, option A is correct.

Q43. Which of the following is true about Glucosinolates:


A. They are the primary source of energy in human diet.
B. They are a group of sulphur and nitrogen containing compounds which contribute to the characteristic’s pungency of
mustard's oil and meal.
C. They are different types of genes introduced through soil bacteria in BT-Brinjal.
D. None of the above.
Answer: B
Explanation:
 Mustard/Rapeseed is one of the most significant oilseed grown in India. But mustard seeds have very high levels
of glucosinolates, a group of sulphur and nitrogen-containing compounds contributing to the characteristic
pungency of their oil and meal.
 This limits the acceptance of mustard oil by many users specially those who are used to less strong odour and
flavour. The problem is even more in case of meal (the residual cake after extraction of oil from the seeds).
 Rapeseed meal is unpalatable to poultry and pigs, while having to be mixed with fodder grass and water for
giving to cattle and buffaloes. Moreover, high glucosinolates are also known to cause goitre (swelling of neck)
and internal organ abnormalities in livestock.
Hence, option B is correct.

Q44. Consider the following statements:


1. As per the current scientific understanding of the Universe, Dark matter makes up more than 50% of the Universe.
2. Dark energy is believed to be the repulsive gravitational effect which pushes the Universe apart.
Which of the above statements is/are correct?
A. 1 only
B. 2 only
C. Both 1 and 2
D. Neither 1 nor 2
Answer: B
Explanation:

2nd Floor, 45 Pusa Road, Opp. Metro Pillar 128, Karol Bagh, New Delhi-110005
Ph: 08045248491, 7041021151 | Email: students@levelupias.com
 Statement 1 is not correct: Dark Matter: Like ordinary matter, dark matter takes up space and holds mass. But it
doesn’t reflect, absorb, or radiate light – at least not enough for us to detect yet. While scientists have measured
that dark matter makes up about 27% of the cosmos, they’re not sure what it is. Theories include several kinds
of as-yet unidentified types of particles that rarely interact with normal matter.
 Statement 2 is correct: Dark Energy may be comprising roughly 68% of the universe, but scientists know even
less about it than they do about dark matter. But something like dark energy must exist to explain the universe’s
accelerating expansion.
o Since the late 1920s, astronomers have known that the universe is expanding. In the 1990s, observations
of distant star explosions, called supernova, showed that the universe expanded more slowly in the past
than it does now. The reason for this remains unclear, but the leading explanation is that the universe
contains something that has a repulsive gravitational effect – it pushes the universe apart instead of
pulling it back together. This phenomenon is called dark energy.
Hence, option B is correct.

Q45. Consider the following statements with reference to Child Adoption in India:
1. The process of adoption in India is monitored and regulated by the Central Adoption Resource Authority (CARA),
which is part of the Ministry of Home Affairs.
2. As per the Adoption Regulations 2022, District Magistrates (DM) including Additional DM, can issue adoption orders.
3. No child shall be given in adoption to a couple unless they have at least two years of stable marital relationship except
in the cases of relative or step-parent adoption.
Which of the above statements is/are correct?
A. 1 and 2 only
B. 1 and 3 only
C. 1, 2 and 3
D. 2 and 3 only
Answer: D
Explanation:
 Statement 1 is not correct: The process of adoption in India is monitored and regulated by the Central Adoption
Resource Authority (CARA) which is a statutory body under Ministry of Women & Child Development. It is the
nodal body for the adoption of Indian children and is mandated to monitor and regulate in-country adoptions.
 Statement 2 is correct: The Adoption Regulations, 2022, introduced various amendments to simplify the
adoption procedure in India under the Juvenile Justice (Care and Protection of Children) Amendment Act, 2021.
One of the key amendments is the authorization for District Magistrates (DMs), including Additional District
Magistrates (ADMs), to issue adoption orders. Previously, adoption orders were issued by courts, but this change
allows for a more streamlined process.
 Statement 3 is correct: Regulation 5 of Adoption Regulations, 2022 provides for the eligibility criteria for
prospective adoptive parents. Regulation 5(3) states that "no child shall be given in adoption to a couple unless
they have at least two years of stable marital relationship except in the cases of relative or step-parent
adoption".

2nd Floor, 45 Pusa Road, Opp. Metro Pillar 128, Karol Bagh, New Delhi-110005
Ph: 08045248491, 7041021151 | Email: students@levelupias.com
Hence, option D is correct.

Q46. Consider the following statements with reference to Denotified Tribes (DNTs) in India:
1. They were originally notified as 'criminal tribes' under the Criminal Tribes Act of 1871, which labelled them as habitual
offenders.
2. Currently, there are only eight denotified tribal communities in India.
3. Bedia, Pardhi, Banjara, and Bahelia are a few examples of denotified tribal communities in India.
4. They often engage in traditional occupations such as hunting, gathering, and performing arts.
How many of the above statements is/are correct?
A. Only one
B. Only two
C. Only three
D. All four
Answer: C
Explanation:
 Statement 1 is correct: The Denotified Tribes (DNTs) were originally notified as "criminal tribes" under the
Criminal Tribes Act of 1871 during British colonial rule in India. This act classified certain tribal communities as
inherently criminal or "habitual offenders" based on their traditional occupations, lifestyles, and mobility
patterns. The colonial administration subjected these communities to surveillance, restrictions on movement,
forced labour, and other forms of discrimination and marginalization.
 Statement 2 is not correct: There are more than 150 Denotified Tribes (DNTs) communities are present in India.
These communities vary widely in terms of culture, language, geographical location, and socio-economic
conditions.
 Statement 3 is correct: Gor, Bedia, Pardhi, Banjara, Bahelia, Sansia, Dom, Kanjar, Bawaria and Sansi are a few
examples of denotified tribal groups.
 Statement 4 is correct: Denotified Tribes (DNTs) in India often engage in traditional occupations such as hunting,
gathering, and performing arts. These occupations have been a part of their cultural heritage and livelihood
practices for generations.
Hence, option C is correct.

Q47. Consider the following statements with reference to the Annual Status of Education Report (ASER) 2023:
1. It is an annual national survey conducted by the Ministry of Education.
2. It is a school-based survey conducted for both rural and urban schools.
3. About 25% of the youth in the age group of 14-18 still cannot read a Class-II level text fluently in their regional language.
4. Across enrollment categories, females do better than males in reading a Class-II level text in their regional language.
How many of the above statements is/are correct?
A. Only one

2nd Floor, 45 Pusa Road, Opp. Metro Pillar 128, Karol Bagh, New Delhi-110005
Ph: 08045248491, 7041021151 | Email: students@levelupias.com
B. Only two
C. Only three
D. All four
Answer: B
Explanation:
 Statement 1 is not correct: Annual Status of Education Report (ASER) 2023 is a citizen-led household survey
conducted by Pratham, a non-governmental organization (NGO) focused on improving educational quality in
India.
o First implemented in 2005, the 'basic’ ASER survey was conducted annually until 2014 and switched to
an alternate-year cycle in 2016. The ‘basic’ ASER collects information about enrollment in pre-school and
school for children in the age group of 3 to 16, and assesses children aged 5 to 16 one-on-one to
understand their foundational reading and arithmetic abilities.
 Statement 2 is not correct: Unlike most other large scale learning assessments, ASER is a nationwide citizen-led
household survey (not school-based) that provides a snapshot of the status of children’s schooling and learning
in rural India.
o The ASER 2023 ‘Beyond Basics’ survey was conducted in 28 districts across 26 states. One rural district
was surveyed in each major state, with the exception of Uttar Pradesh and Madhya Pradesh, where
two rural districts were surveyed.
 Statement 3 is correct: About 25% of the youth in the age group of 14-18 still cannot read a Class-II level text
fluently in their regional language. More than half struggle with division (3-digit by 1-digit) problems. Only 43.3%
of 14-18-year-olds are able to do such problems correctly. This skill is usually expected in Std III/IV. A little over
half can read sentences in English (57.3%). Of those who can read sentences in English, almost three quarters can
tell their meanings (73.5%).
 Statement 4 is correct: Across enrollment categories, females (76%) do better than males (70.9%) in reading a
Class-II level text in their regional language. In contrast, males do better than their female counterparts in
arithmetic and English reading.
Hence, option B is correct.

Q48. Consider the following statements with reference to World Malaria Report 2023:
1. It is released by the World Health Organization (WHO).
2. India accounted for the highest number of malaria cases globally in 2022.
3. Climate change is considered a major driver affecting malaria transmission.
4. Globally, there is a decline in numbers of malaria cases in 2022 as compared to the previous year.
Which of the statements given above is/are correct?
A. 1, 2, 3 and 4
B. 2 and 4 only
C. 3 only
D. 1 and 3 only

2nd Floor, 45 Pusa Road, Opp. Metro Pillar 128, Karol Bagh, New Delhi-110005
Ph: 08045248491, 7041021151 | Email: students@levelupias.com
Answer: D
Explanation:
 Statement 1 is correct: The World Malaria Report is an annual publication released by the World Health
Organization (WHO) to provide an update on the global malaria situation, including key statistics, trends, and
challenges in malaria control and elimination efforts.
 Statement 2 is not correct: Four countries, Nigeria (27%), the Democratic Republic of the Congo (12%), Uganda
(5%), and Mozambique (4%), accounted for almost half of all malaria cases globally.
o India accounted for a significant 66% of malaria cases in South-East Asia Region, but it global
contribution is 1.4% only.
 Statement 3 is correct: Climate change is considered a major driver affecting malaria transmission. Changing
climate conditions can influence the distribution and behavior of mosquitoes that transmit malaria, as well as
the development and survival of the malaria parasite within mosquitoes and humans. This can lead to changes in
malaria transmission patterns and the overall burden of the disease.
 Statement 4 is not correct: Report reveals a global surge in malaria cases. There were 5 million additional malaria
cases in 2022 as compared to the previous year, totaling to 249 million.
o Of the 5 million additional cases, the highest 2.1 million was from Pakistan mainly because of 2022 flood.
Hence, option D is correct.

Q49. Recently, the term ‘Grey zone warfare’ was in news. It refers to which of the following?
A. It is a strategy where a country tries to gain an advantage without openly going to war.
B. It is divide-and-conquer tactic used to dominate opposition territory, piece by piece.
C. It refers to geopolitical initiative to build a network of one’s state-owned ports and maritime infrastructure.
D. It is a military strategy to control access to and within an operating environment.
Answer: A
Explanation:
 Option A is correct: Grey zone warfare generally means a middle, unclear space that exists between direct conflict
and peace in international relations. It is a strategy where a country tries to gain an advantage without openly
going to war. It involves tactics like cyberattacks, misinformation, and economic pressure to weaken an opponent.
o On the last day of the 2024 Raisina Dialogue (February 24), India’s Chief of Defence Staff, said that “grey
zone warfare” is the latest in informal warfare.
 Option B is not correct: Salami Slicing strategy refers to a divide-and-conquer tactic used to dominate opposition
territory, piece by piece. It involves incremental threats and alliances to overcome opposition and acquire new
territories.
 Option C is not correct: String of Pearls strategy refers to a geopolitical and strategic initiative that involves
building a network of Chinese-funded, owned, or controlled ports and other maritime infrastructure facilities in
strategic locations across the Indian Ocean.
 Option D is not correct: Anti-access/Area Denial (A2/AD) strategy is a military strategy to control access to and
within an operating environment.

2nd Floor, 45 Pusa Road, Opp. Metro Pillar 128, Karol Bagh, New Delhi-110005
Ph: 08045248491, 7041021151 | Email: students@levelupias.com
o For example, China is employing a multi-faceted A2/AD strategy to exclude or deter as many adversary
forces from acting within the waters it claims as its territorial waters as outlined by the nine-dash line.
Hence, option A is correct.

Q50. Consider the following statements with reference to the North Atlantic Treaty Organization (NATO):
1. It was established by the Washington Treaty to provide security against the Soviet Union.
2. It is headquartered in Washington, USA.
3. Its protection does not extend to members' civil wars or internal coups.
4. Its decisions are taken by the consensus of all the 32 members of the organisation.
How many of the above statements is/are correct?
A. Only one
B. Only two
C. Only three
D. All four
Answer: C
Explanation:
 Statement 1 is correct: The North Atlantic Treaty Organization (NATO) is a military alliance established by the
North Atlantic Treaty (also called the Washington Treaty) of 4th April, 1949, by the United States, Canada, and
several Western European nations to provide collective security against the Soviet Union.
 Statement 2 is not correct: NATO is headquartered in Brussels, Belgium (not in Washington, USA)
 Statement 3 is correct: NATO's protection is based on the principle of collective defense, as outlined in Article 5
of the NATO treaty. However, this protection does not extend to members' civil wars or internal coups. It is
meant to respond to attacks against any NATO member state.
 Statement 4 is correct: Every day, member countries consult and take decisions on security issues at all levels and
in a variety of fields. A “NATO decision” is the expression of the collective will of all 32 member countries since
all decisions are taken by consensus.
NOTE: There was a typing error in the question paper in Statement 4 that we have corrected in this solution file.
Hence, option C is correct.

Q51. Consider the following statement with reference to the Lake Chad:
1. It is bordered by 8 countries.
2. The Chari River, originating from the Central African Republic and flowing through Chad into Lake Chad, contributes
90% of the water that enters Lake Chad.
3. As per the recent report, the lake has dried up completely.
How many of the above statements is/are correct?
A. Only one
B. Only two

2nd Floor, 45 Pusa Road, Opp. Metro Pillar 128, Karol Bagh, New Delhi-110005
Ph: 08045248491, 7041021151 | Email: students@levelupias.com
C. All three
D. None
Answer: A
Explanation:
 Statement 1 is not correct: Lake Chad is
located in the Sahel, the vast semi-arid
region south of the Sahara desert. It is
bordered by 4 countries - Chad,
Cameroon, Niger and Nigeria.
 Statement 2 is correct: Lake Chad is fed
mainly by the Chari River through the
Lagone tributary. The Lagone tributary
used to provide 90 % of its water. The Lake
is vital for indigenous, pastoral and farming
communities in these countries.
 Statement 3 is not correct: Lake Chad has
not dried up completely, but it has shrunk
by 90% since the 1960s, from 25,000
square kilometres in 1963 to less than
1,500 square kilometres in 2001. The lake's
decline is due to climate change,
population pressures, and overuse of
resources.
Hence, option A is correct.

Q52. Consider the following statements with reference to the Gulf of Aqaba:
1. It is a northeastern arm of the Red Sea located between the Sinai Peninsula to the west and the Arabian Peninsula to
the east.
2. It is bordered by 2 countries.
3. It is a part of the Great Rift Valley system and is formed by the tectonic movements.
How many of the above statements is/are correct?
A. Only one
B. Only two
C. All three

2nd Floor, 45 Pusa Road, Opp. Metro Pillar 128, Karol Bagh, New Delhi-110005
Ph: 08045248491, 7041021151 | Email: students@levelupias.com
D. None
Answer: B
Explanation:
 Statement 1 is correct: The Gulf of Aqaba is a large
gulf at the northern tip of the Red Sea, located
between the Sinai Peninsula to the west and the
Arabian Peninsula to the east. The Gulf of Aqaba is
also known as the Gulf of Eilat. It is 110 miles (177 km)
long and varies in width from 12 to 17 miles (19 to 27
km).
 Statement 2 is not correct: The Gulf of Aqaba is part
of the complex East African Rift System, and it is
bordered by 4 countries - Egypt, Israel, Jordan, and
Saudi Arabia.
 Statement 3 is correct: The Gulf of Aqaba is part of
the Great Rift Valley system, which is a vast
geological feature extending from the Levant region
in the north to Mozambique in the south. This system
is characterized by a series of rift valleys, fault lines,
and geological formations resulting from the
movement of tectonic plates.
Hence, option B is correct.

Q53. Consider the following statements about Gulf of Guinea:


1. It is located on the West Coast of Australia.
2. The Zero-degree longitude and zero-degree latitude passes through it.
3. Sao Tome and Principe are small island in the Gulf of Guinea.
Which of the above statements is/are correct?
A. 1 only
B. 2 and 3 only
C. 1 and 3 only
D. 1, 2 and 3
Answer: B
Explanation:

2nd Floor, 45 Pusa Road, Opp. Metro Pillar 128, Karol Bagh, New Delhi-110005
Ph: 08045248491, 7041021151 | Email: students@levelupias.com
 Statement 1 is not correct: The Gulf
of Guinea is situated on the western
coast of Africa, stretching from Cape
Lopez in Gabon to Cape Palmas in
Liberia. It is an integral part of the
Atlantic Ocean and is bordered by
countries such as Nigeria, Cameroon,
Equatorial Guinea, and Ghana,
among others.
 Statement 2 is correct: The
intersection of the Prime Meridian (0°
longitude) and the Equator (0°
latitude) falls within the Gulf of
Guinea. This point is also known as
‘Null Island.’
 Statement 3 is correct: Sao Tome and
Principe are small island nation
located entirely within the Gulf of
Guinea.

Hence, option B is correct.

Q54. Consider the following statements with respect to Automated Permanent Academic Account Registry (APAAR) ID:
1. It is envisioned as a special ID system for all students in India starting from childhood.
2. It is part of the ‘One nation, One Student ID’ initiative stemming from the New Education Policy (NEP) 2020.
3. Only for the students switching schools within the same state, all related data stored in the Academic Bank of Credits
(ABC) gets transferred to the new school by sharing the APAAR ID.
How many of the above statements is/are correct?
A. Only one
B. Only two
C. All three
D. None
Answer: B
Explanation:
 Statement 1 is correct: It is envisioned as a special ID system for all students in India starting from childhood.
Under the initiative, each student would get a lifelong APAAR Id, making it easy for the learners, schools and
governments to track academic progress from pre-primary education to higher education.

2nd Floor, 45 Pusa Road, Opp. Metro Pillar 128, Karol Bagh, New Delhi-110005
Ph: 08045248491, 7041021151 | Email: students@levelupias.com
 Statement 2 is correct: It is part of the ‘One nation, One Student ID’ initiative stemming from the New Education
Policy (NEP) 2020. With the APAAR ID, students would be able to store all their certificates and credits, whether
they come from formal education or informal learning. It would also serve as gateway to Digilocker.
 Statement 3 is not correct: If the student changes schools, whether within the state or to another state, all
related data in the Academic Bank of Credits gets transferred to the new school just by sharing the APAAR ID.
Hence, option B is correct.

Q55. Consider the following statements about e-Jagriti PORTAL:


1. It offers an efficient, speedy, and economical software solution for resolving consumer disputes across all levels.
2. It offers modules for case filing, online fee payment, and case monitoring, facilitating the swift disposal of cases by all
commissions.
3. The portal also includes a Virtual court facility, providing consumers with a convenient and accessible means of
resolving their complaints.
4. It is an initiative of the Ministry of Commerce and Industry.
How many of the above statements is/are correct?
A. Only one
B. Only two
C. Only three
D. All four
Answer: C
Explanation:
 Statement 1 is correct: The e-Jagriti Portal is a software solution that provides a simple, fast, and cost-effective
way to resolve consumer disputes at all levels. The portal integrates various consumer grievance platforms into
a single platform, including the Online Case Monitoring System (OCMS), E-Daakhil, NCDRC Case Monitoring
System, and the CONFONET website.
 Statement 2 is correct: The e-Jagriti platform has case filing, online fee payment, case monitoring modules for
seamless disposal of cases by all the Commissions, has Smart search facility on archived consumer
complaints/cases/judgements using AI technology for metadata and keyword creation, and Voice-to-text
conversion of judgements, case history and other details using AI/ML technology.
 Statement 3 is correct: The portal will integrate a Virtual court facility for a convenient and accessible resolution
of consumer complaints, reducing the time of disposal, multiple hearings, and physical court appearances,
bringing effective and fast decisions & disposals in all Consumer Commissions.
 Statement 4 is not correct: It is an initiative of the Department of Consumer Affairs, Ministry of Consumer
Affairs, Food, and Public Distribution.
Hence, option C is correct.

2nd Floor, 45 Pusa Road, Opp. Metro Pillar 128, Karol Bagh, New Delhi-110005
Ph: 08045248491, 7041021151 | Email: students@levelupias.com
Q56. Consider the following statements with respect to Project Akashteer:
1. It is an initiative developed by Defence Research and Development Organisation (DRDO) under the 'Atmanirbhar
Bharat' scheme.
2. It aims to automate air defense control and reporting processes through digitization.
3. It will enable monitoring of low-level airspace over the battle areas of Indian Army.
How many of the above statements is/are correct?
A. Only one
B. Only two
C. All three
D. None
Answer: B
Explanation:
 Statement 1 is not correct: Project Akashteer is an initiative developed by Bharat Electronics Limited (BEL) (not
DRDO) under the 'Atmanirbhar Bharat' scheme. BEL is a Navratna PSU under the Ministry of Defence.
 Statement 2 is correct: It is a cutting-edge initiative designed to automate air defense control and reporting
processes by digitizing them. It aims to deliver an unprecedented level of situational awareness and control for
the force to ensure the safety of friendly aircraft and engage hostile aircraft in contested airspace.
 Statement 3 is correct: It will enable monitoring of low-level airspace over the battle areas of Indian Army and
effectively control the Ground Based Air Defence Weapon Systems.
Hence, option B is correct.

Q57. Which Indian folk-dance form is characterized by vigorous movements, rhythmic clapping, and the use of traditional
musical instruments like the dhol and tumbi?
A. Bhangra
B. Kathakali
C. Bihu
D. Garba
Answer: A
Explanation:
Bhangra:
 Bhangra is a type of traditional folk-dance from the Indian subcontinent, originating in the Majha area of Punjab.
The dance was associated primarily with the spring harvest festival, Baisakhi.
 It is characterized by vigorous movements, rhythmic clapping, and the use of traditional musical instruments like
the dhol (drum) and tumbi (single-stringed instrument).
 It features dynamic footwork, lively music, and distinctive costumes, including colorful turbans.
Hence, option A is correct.

2nd Floor, 45 Pusa Road, Opp. Metro Pillar 128, Karol Bagh, New Delhi-110005
Ph: 08045248491, 7041021151 | Email: students@levelupias.com
Q58. Consider the following statements with respect to Guruvayur Temple:
1. It is a Hindu temple located in Tamil Nadu, often referred to as the ‘Dwarka of the south’.
2. It is dedicated to Lord Shiva.
3. It has structures like the Nalambalam (temple structure surrounding the sanctum sanctorum), Balikkal (sacrificial
stone) and Deepastambam (pillar of lights).
How many of the above statements is/are correct?
A. Only one
B. Only two
C. All three
D. None
Answer: A
Explanation:
 Statement 1 is not correct: It is a Hindu temple, often referred to as the Southern Dwarka. It is located in the
town of Guruvayur in Kerala (not Tamil Nadu).
 Statement 2 is not correct: It is dedicated to Lord Vishnu in the youthful form of Lord Krishna. The Chief deity
here is Lord Krishna, also known as Guruvayoorappan. The chief deity is portrayed standing with four hands,
grasping Shankh, Sudarshana Chakra, Lotus, and Mace, all carved from a stone known as Patala Anjana.
 Statement 3 is correct: It follows the traditional Kerala architectural style. It has structures like the Nalambalam
(temple structure surrounding the sanctum sanctorum), Balikkal (sacrificial stone) and Deepastambam (pillar of
lights). A prominent feature is the Dwajasthamba, a 70-foot-tall flagstaff completely coated in gold.
Additional Info:
 In 1931-32, Satyagraha was initiated by Kelappan (known as Kerala Gandhi) to advocate for the inclusion of
untouchables into the temple. It resulted in the Travancore Temple Entry Proclamation of 1936.
Hence, option A is correct.

Q59. Consider the following statements:


1. The ‘Theras’ were the chief preceptors of the Jain order after Mahavira.
2. The ‘Purvas’ are the doctrinal texts of Jainism, which contained the original doctrines of Mahavira.
3. ‘Parishishtaparvan’ is a famous Buddhist text, consists of the history of the earliest Buddhist teachers.
4. ‘Yapaniya’ is a Buddhist sect.
How many of the above statements is/are correct?
A. Only one
B. Only two
C. Only three
D. All four
Answer: B
Explanation:

2nd Floor, 45 Pusa Road, Opp. Metro Pillar 128, Karol Bagh, New Delhi-110005
Ph: 08045248491, 7041021151 | Email: students@levelupias.com
 Statement 1 is correct: Mahavira had 11 disciples, known as the Ganadharas or heads of the schools. Arya
Sudharma was the sole Ganadhara who survived Mahavira and became the first 'Thera' (the chief preceptor) of
the Jaina order. He passed away 20 years after Mahavira's demise. During the reign of the late Nanda king, the
Jain order was overseen by two Theras:
o Sambhutavijaya
o Bhadrabahu
 Statement 2 is correct: According to tradition, Mahavira's original teachings were preserved in 14 ancient texts
called the 'Purvas'.
o At the first Council in Pataliputra, Sthulabhadra organized the Jaina Canon into 12 'Angas' or sections, a
structure accepted by the Svetambaras. However, the Digambaras disagreed, claiming the original
scriptures were lost.
o The second Council at Vallabhi saw new additions known as 'Upangas' or minor sections. Among the 12
Angas, the Acharanga Sutra focuses on the conduct of Jaina monks, while the Bhagavati Sutra
comprehensively explains Jaina doctrines.
 Statement 3 is not correct: 'Parishishtaparvan' is a significant Jain text that focuses on the history of the earliest
Jain teachers, known as Tirthankaras. It provides details about their lives, teachings, and the early Jain
community. It is part of the Jain Agamas, the canonical literature of Jainism.
 Statement 4 is not correct: ‘Yapaniya’ is a Jain order that existed in western Karnataka, particularly in the regions
of modern-day Dharwad and Belgaum. It was known for its adherence to strict principles of asceticism and
monastic discipline. Over time, the Yapaniya order declined and is now considered extinct.
Hence, option B is correct.

Q60. Consider the following statements with reference to the Non-Cooperation Movement:
1. Gandhi’s proposal to start the Non-Cooperation Movement received unanimous support from the fellow Congressmen.
2. The Nagpur Session of the Congress accepted the mandate that the Non-Cooperation Movement will be implemented
by the All-India Congress Committee.
3. The Central Khilafat Committee endorsed the Non-Cooperation Movement after the Indian National Congress.
4. Lokmanya Tilak passed away on the day the movement was launched.
How many of the above statements is/are correct?
A. Only one
B. Only two
C. Only three
D. All four
Answer: B
Explanation:
 Statement 1 is not correct: While Gandhi was highly influential within the Indian National Congress, not all
Congress members unanimously supported his proposal for the Non-Cooperation Movement. There were
debates and differing opinions within the Congress regarding the movement's strategy, timing, and potential

2nd Floor, 45 Pusa Road, Opp. Metro Pillar 128, Karol Bagh, New Delhi-110005
Ph: 08045248491, 7041021151 | Email: students@levelupias.com
consequences. Some Congress leaders, like Motilal Nehru and Chittaranjan Das, initially had reservations about
the movement's radical nature and its potential to incite violence.
 Statement 2 is correct: The Nagpur Session of the Congress, held in December 1920, endorsed the Non-
Cooperation Programme. This programme included the mandate that the movement would be implemented in
stages, as directed by the All-India Congress Committee. This decision was an important step in formalizing the
structure and strategy of the Non-Cooperation Movement within the Congress framework.
 Statement 3 is not correct: The Central Khilafat Committee endorsed the Non-Cooperation Movement first, at
the Allahabad Conference held on 1st - 2nd June 1920. The decision to launch the movement was made at this
conference, before the endorsement by the Indian National Congress. The Khilafat Movement, which aimed to
support the Ottoman Caliphate, was closely aligned with the Indian independence movement, and their
endorsement was significant in mobilizing Muslim support for the Non-Cooperation Movement.
 Statement 4 is correct: Lokmanya Bal Gangadhar Tilak, a prominent leader in the Indian independence
movement, passed away on August 1, 1920. This was the same day that Mahatma Gandhi launched the Non-
Cooperation Movement. In honor of Tilak's legacy, Gandhi named the movement's fund the Tilak Swaraj Fund,
which received overwhelming support and contributions from the public.
Hence, option B is correct.

Q61. Consider the following responses of nationalists to the British participation in the first world war:
1. The moderates supported the empire as a matter of duty.
2. The Revolutionaries supported the war in a mistaken belief that Britain would repay India's loyalty with gratitude.
3. The Extremists decided to utilize the opportunity to wage war on Britain's rule and liberate the country.
How many of the above statements is/are correct?
A. Only one
B. Only two
C. All three
D. None
Answer: A
Explanation:
 Statement 1 is correct: The Moderates were a group within the Indian National Congress who believed in
peaceful and gradual reforms under British rule. They supported the British Empire's participation in the war
because they felt it was their duty to support the empire in its time of need. They believed that by supporting
the war effort, India would gain goodwill from Britain, which could eventually lead to greater self-governance
or reforms within the British administration in India.
 Statement 2 is not correct: The Extremists, led by Bal Gangadhar Tilak and Bipin Chandra Pal, were another
faction within the Indian National Congress. They were more radical in their approach. During the First World
War, they supported the British war efforts, believing that India's loyalty and contribution to the war would
lead to significant political concessions from the British, including self-government or independence. However,
this belief was mistaken, as Britain did not fulfill these promises after the war.

2nd Floor, 45 Pusa Road, Opp. Metro Pillar 128, Karol Bagh, New Delhi-110005
Ph: 08045248491, 7041021151 | Email: students@levelupias.com
 Statement 3 is not correct: The revolutionaries, also known as the militant nationalists, were a group of activists
who believed in using armed resistance and violent means to overthrow British rule in India. They saw the First
World War as an opportunity to weaken British power and launched several revolutionary activities during this
period, including uprisings and acts of violence against British officials. They hoped that by taking advantage of
Britain's distraction with the war, they could achieve their goal of liberating India from British rule.
Hence, option A is correct.

Q62. Consider the following statements with reference to V D Savarkar:


1. He was the member of India house that was started in 1905 in London by Shyamji Krishnaverma.
2. He founded the Indian Independence league.
3. He, via his book, became the first writer to call 1857 revolt as India’s first war for independence against the British.
4. ‘Six glorious epochs of Indian history’ is a book written by V D Savarkar.
How many of the above statements is/are correct?
A. Only one
B. Only two
C. Only three
D. All four
Answer: C
Explanation:
Vinayak Damodar Savarkar, commonly known as Veer Savarkar, was a prominent Indian nationalist, freedom fighter,
and political leader. He was born on 28th May 1883 in a village near Nashik, Maharashtra. Savarkar is known for his
contributions to the Indian independence movement and his ideology of Hindutva, which emphasized the cultural and
religious unity of Hindus.
 Statement 1 is correct: Vinayak Damodar Savarkar was a member of India House in London. India House was a
student residence and center for Indian nationalist activities, founded by Shyamji Krishnavarma in 1905. It
became a hub for Indian students and nationalists living in Britain at the time, including many prominent figures
in the Indian independence movement.
 Statement 2 is not correct: The Indian Independence League was a Political Organization that was operated from
the 1920s to the 1940s. It was founded by Rash Behari Bose during a conference in Tokyo convened by him. It
was set up to organize those living outside India into seeking the removal of British rule from India.
o After the arrival of Subhas Chandra Bose in South East Asia and the revamp of the Indian National Army,
league came under his leadership.
 Statement 3 is correct: Savarkar wrote a book titled 'The History of the War of Indian Independence' in which
he referred to the 1857 revolt as India's first war for independence against the British. This was a significant
reinterpretation of the events of 1857, as prior to this, the revolt was often referred to as a mere mutiny or sepoy
rebellion.

2nd Floor, 45 Pusa Road, Opp. Metro Pillar 128, Karol Bagh, New Delhi-110005
Ph: 08045248491, 7041021151 | Email: students@levelupias.com
 Statement 4 is correct: 'Six Glorious Epochs of Indian History' is one of the books written by V D Savarkar. In this
book, he discusses various important periods in Indian history which he considers to be glorious epochs,
including ancient Indian civilization, the Maurya Empire, the Maratha Empire, and others.
Hence, option C is correct.

Q63. Consider the following statements:


Statement-I:
During the period of British expansion in Asia, there was a significant drain of wealth from India.
Statement-II:
The British utilized the Indian army for their military campaigns and relied on high taxation to finance their military
operations.
Which one of the following is correct in respect of the above statements?
A. Both Statement-I and Statement-II are correct and Statement-II is the correct explanation for Statement-I.
B. Both Statement-I and Statement-II are correct and Statement-II is not the correct explanation for Statement-I.
C. Statement-I is correct but Statement-II is incorrect.
D. Statement-I is incorrect but Statement-II is correct.
Answer: A
Explanation:
 Statement-I is correct: During the British expansion in Asia, there was a significant drain of wealth from India. It
was highlighted by Dadabhai Naoroji's 'drain of wealth' theory, which he articulated in his book ‘Poverty and
Un-British Rule in India’. He argued that British colonial rule was exploiting India economically, leading to a drain
of wealth. He identified several factors contributing to this drain, including the payment of 'home charges' to
cover the costs of the British civil administration and military in India, the lack of immigration to India which
would bring labor and capital for economic growth, and the exploitation of India's resources and markets for
the benefit of Britain.
 Statement-II is correct: The British utilize the Indian army for their military campaigns, both within India and in
other parts of the world where the British Empire was expanding. The maintenance of a large standing army,
along with other aspects of British imperial policy, required significant financial resources. To finance these
operations, the British relied in part on high taxation in India, which contributed to the economic burden on
the Indian population and the drain of wealth from the country.
Hence, option A is correct.

Q64. Which of the following statements is/are correct regarding the Indigo Revolt of 1859?
1. The intelligentsia’s role in the Indigo Revolt was to have an abiding impact on the emerging nationalist intellectuals.
2. The Hindu Patriot played an important role in this movement.
3. Dinabandhu Mitra’s play, “Neel Darpan”, portrayed the oppression by the indigo planters.
Select the correct answer using the code given below:
A. 1 only

2nd Floor, 45 Pusa Road, Opp. Metro Pillar 128, Karol Bagh, New Delhi-110005
Ph: 08045248491, 7041021151 | Email: students@levelupias.com
B. 2 and 3 only
C. 3 only
D. 1, 2 and 3
Answer: D
Explanation:
 Statement 1 is correct: During the Indigo Revolt, the intelligentsia of Bengal played a significant role in
supporting the rebellious peasants. They organized a powerful campaign that included activities such as
carrying out newspaper campaigns, organizing mass meetings, preparing memoranda on the peasants’
grievances, and supporting them in their legal battles. This active involvement of the intelligentsia was not just
a short-term contribution to the revolt but had a lasting impact. It influenced and inspired emerging nationalist
intellectuals, highlighting the power of collective action against oppression and contributing to the growth of
nationalist sentiments in India.
 Statement 2 is correct: The Hindu Patriot, a newspaper edited by Harish Chandra Mukherji, played a crucial role
in the Indigo Revolt. Mukherji published regular reports from his correspondents in rural areas, which highlighted
the oppression faced by the peasants at the hands of the indigo planters. These reports helped to raise
awareness about the plight of the peasants and the injustices they were facing. Additionally, Mukherji wrote
passionately and knowledgeably about the issue, elevating it to a high political level. The newspaper's coverage
and advocacy contributed significantly to the visibility and support for the peasant movement.
 Statement 3 is correct: Dinabandhu Mitra's play, "Neel Darpan" (The Indigo Mirror), became famous for vividly
portraying the oppression faced by the peasants under the indigo planters. The play depicted the harsh and
exploitative practices of the planters, highlighting the suffering of the peasants who were forced to grow indigo
against their will. Through its powerful portrayal of the peasants' struggles, the play not only raised awareness
about the issue but also contributed to the broader discourse on colonial oppression and exploitation in India.
Hence, option D is correct.

Q65. Consider the following are the powers and functions of the Election Commission of India:
1. To advise the President on the matters relating to the disqualification of the members of the Parliament.
2. To appoint officers for inquiring into the disputes relating to the electoral arrangements.
3. To supervise the machinery of the elections to the Panchayats and the municipalities in the states.
4. To grant recognition to the political parties and allot election symbols to them.
How many of the above statements is/are correct?
A. Only one
B. Only two
C. Only three
D. All four
Answer: C
Explanation:

2nd Floor, 45 Pusa Road, Opp. Metro Pillar 128, Karol Bagh, New Delhi-110005
Ph: 08045248491, 7041021151 | Email: students@levelupias.com
 Statement 1 is correct: The Election Commission of India is responsible for advising the President on matters
related to disqualification of members of Parliament. This advice is typically given when there are allegations or
instances of misconduct by members of Parliament that could lead to disqualification under the Constitution of
India or any relevant laws.
 Statement 2 is correct: The Election Commission has the authority to appoint officers who investigate and
resolve disputes related to electoral arrangements. These disputes could include issues such as malpractices
during elections, irregularities in voter lists, or any other matters that affect the conduct of free and fair elections.
These officers appointed by the Election Commission assist in ensuring the integrity and transparency of the
electoral process.
 Statement 3 is not correct: The ECI is responsible for conducting and supervising elections for Parliament, state
legislatures, President, and Vice President.
o Local body elections, including Panchayats and municipalities, fall under the purview of State Election
Commissions. While the ECI might provide guidance and advice, it doesn't directly manage these
elections.
 Statement 4 is correct: The Election Commission is responsible for granting recognition to political parties and
allocating unique symbols to them for contesting elections. This recognition is based on certain criteria, including
the party's performance in past elections. Allotment of symbols is crucial as it helps voters identify and
differentiate between various political parties and independent candidates during elections.
Hence, option C is correct.

Q66. Consider the following statements:


1. Religious instructions shall be provided in any educational institution wholly maintained out of the state funds.
2. Religious instructions cannot be provided in any educational institution administered by the state, but established under
any endowment or trust.
3. Religious instruction can be permitted on a voluntary basis in educational institutions recognized by the state or
receiving aid out of the state funds.
How many of the above statements is/are correct?
A. Only one
B. Only two
C. All three
D. None
Answer: A
Explanation:
 Statement 1 is not correct: According to Article 28, religious instruction shall not be provided in any educational
institution wholly maintained out of state funds. The emphasis here is on the complete prohibition of religious
instruction in institutions funded entirely by the state.
 Statement 2 is not correct: Article 28 makes a distinction for educational institutions administered by the state
but established under any endowment or trust. In such cases, religious instruction is permitted. The key point

2nd Floor, 45 Pusa Road, Opp. Metro Pillar 128, Karol Bagh, New Delhi-110005
Ph: 08045248491, 7041021151 | Email: students@levelupias.com
is that if the institution is administered by the state but has been established under an endowment or trust that
requires the imparting of religious instruction, then such instruction is allowed.
 Statement 3 is correct: Article 28 allows for religious instruction on a voluntary basis in educational institutions
recognized by the state or receiving aid out of state funds. However, it's important to note that it is not
mandatory. The provision emphasizes the voluntary nature of religious instruction, meaning that no student can
be compelled to attend such instruction against their will. Additionally, in the case of minors, the consent of their
guardian is required.
Hence, option A is correct.

Q67. 'Equal Protection of the Laws' in the India Constitution is:


A. Equal treatment in equal circumstances.
B. Equal subjection of all classes to the ordinary law.
C. Absence of privileges.
D. All the laws are equal and need to be enforced with the same rigour by the administration.
Answer: A
Explanation
Equal Protection of the Laws:
 Article 14 of the Indian Constitution stipulates that the ‘state shall not deny to any person equality before law
or the equal protection of the laws within the territory of India.’
 Although the phrases "equal protection of the laws" and "equality before law" may appear similar, they hold
distinct meanings.
 While equality before the law signifies the absence of any special privilege based on factors such as birth or
religion and the equal application of laws to all classes, equal protection of the laws embodies a more proactive
notion, implying the right to equal treatment under similar circumstances.
 Equal protection of the laws implies that similar cases should be treated similarly, ensuring fairness and justice
in the legal system.
Hence, option A is correct.

Q68. Which of the following organizations has/have been exempted under the Right to Information Act, 2005?
1. Intelligence Bureau
2. National Investigation Agency
3. Narcotics Control Bureau
4. Central Vigilance Commission
5. Directorate of Enforcement
Select the correct answer using the code given below:
A. 1 and 4 only
B. 4 and 5 only

2nd Floor, 45 Pusa Road, Opp. Metro Pillar 128, Karol Bagh, New Delhi-110005
Ph: 08045248491, 7041021151 | Email: students@levelupias.com
C. 2 and 3 only
D. 1, 2, 3 and 5 only
Answer: D
Explanation:
 As per Section 24 of the RTI Act, under Schedule 2, there are intelligence and security organizations which have
been exempted from the purview of the RTI Act. Intelligence Bureau, National Investigation Agency, Directorate
of Enforcement and Narcotics Control Bureau are not under the purview of RTI Act.
 However, the Central Vigilance Commission (CVC) is under the purview of RTI.
Hence, option D is correct.

Q69. Consider the following statements with reference to the Political parties in India:
1. New political parties are registered under the Representation of the People Act, of 1951.
2. A party can be recognised as a National Party if it secures 2% of Lok Sabha seats from a minimum of three different
States.
3. Both recognized and unrecognized political parties are entitled to collect donations that are fully exempted from income
tax.
How many of the above statements is/are correct?
A. Only one
B. Only two
C. All three
D. None
Answer: C
Explanation:
 Statement 1 is correct: To establish a political party, an association or group of individuals must register with the
Election Commission of India under Section 29A of the Representation of the People Act, 1951. Following
registration, a party can be classified as either recognized or unrecognized.
 Statement 2 is correct: According to the Election Symbols (Reservation and Allotment) Order, 1968, a political
party can be recognized as a National Party if:
o It is ‘recognised’ in four or more states as a state party; or
o If its candidates polled at least 6% of total valid votes in any four or more states in the last Lok Sabha or
Assembly elections and has at least four MPs in the last Lok Sabha polls; or
o If it has won at least 2% of the total seats in the Lok Sabha from not less than three states.
 Statement 3 is correct: All registered parties, whether recognized or unrecognized, are eligible to collect
donations that are fully exempt from income tax.
o Recognized political parties are entitled to reserve a symbol for exclusive use, while unrecognized parties
must select from a list of free symbols. Recognized parties also receive additional benefits, such as free
broadcast facilities on Doordarshan and All India Radio, increased allowances for campaign expenditure,
and complimentary copies of electoral rolls before elections.

2nd Floor, 45 Pusa Road, Opp. Metro Pillar 128, Karol Bagh, New Delhi-110005
Ph: 08045248491, 7041021151 | Email: students@levelupias.com
Hence, option C is correct.

Q70. Which of the following best describes about the writ of Habeas Corpus?
A. The writ is in the nature of a proceeding whereby the court enquires into the legality of the claim which a party asserts
to a public office and to oust him from its enjoyment if the claim is not well founded.
B. The writ demands some activity on the part of the body or person to whom it is addressed and commands the person
to whom it is addressed to perform certain public or quasi-legal public duty which he has refused to perform and the
performance of which cannot be enforced by any other adequate legal remedy.
C. The writ is issued by the High Court or the Supreme Court to a lower court forbidding the latter to continue
proceedings therein in excess of its jurisdiction with which it is not legally vested.
D. The writ is in the nature of an order calling upon the state who has detained another person to produce before the
court in order to let the court know on what ground the person has been detained or confined and to set him free if
there is no legal justification for the imprisonment.
Answer: D
Explanation:
 Option (A) is not correct: Quo warranto: This is a legal proceeding that challenges the right of someone to hold
a public office. It essentially asks the court to determine if the person was rightfully appointed or elected to the
position. If the court finds the claim to the office invalid, it can issue an order removing the person from the
position.
 Option (B) is not correct: Mandamus: This writ is a court order that compels a public official or government body
to perform a specific legal duty that they are neglecting or refusing to fulfill. It's used in situations where there's
a clear legal obligation for the official to act, but they haven't done so.
 Option (C) is not correct: Prohibition: This writ is issued by a higher court to a lower court or tribunal, prohibiting
them from exceeding their jurisdiction. It acts as a check on lower courts to ensure they stay within the legal
boundaries of their authority.
 Option (D) is correct: Habeas Corpus: It is an order that compels the state or detaining authority to present a
detained person before the court. The term "Habeas Corpus" is Latin for "you shall have the body," which
essentially means that the court orders the detaining authority to bring the detained person before the court and
justify their detention. This allows the court to review the legality of the detention and, if there is no legal
justification, to order the person's release.
Hence, option D is correct.

2nd Floor, 45 Pusa Road, Opp. Metro Pillar 128, Karol Bagh, New Delhi-110005
Ph: 08045248491, 7041021151 | Email: students@levelupias.com
Q71. Consider the following statements with reference to the Inter-State Council:
1. The Inter-State Council has been established by an executive notification.
2. The Inter-State Council has the power to inquire into and advice upon disputes which may have arisen between the
States.
3. The Prime Minister is the Chairman of the Inter-State Council.
How many of the above statements is/are correct?
A. Only one
B. Only two
C. All three
D. None
Answer: B
Explanation:
 Statement 1 is correct: Inter-State Council is a non-permanent constitutional body set up by a presidential order
on the basis of provisions in Article 263. The body was formed by a presidential order dated 28th May 1990 on
the recommendation of Sarkaria Commission.
 Statement 2 is not correct: The Inter-State Council does not have the power to inquire into and advise upon
disputes between States.
o The Council's functions are mainly to investigate and discuss subjects of common interest among the
States and the Union, make recommendations for better coordination of policy and action, and
deliberate on matters referred by the Chairman (who is the Prime Minister).
 Statement 3 is correct: The Prime Minister serves as the Chairman of the Inter-State Council, as per the
Presidential Order. The Council also includes the Chief Ministers of all States, the Chief Ministers of Union
Territories with Legislative Assemblies, and other nominated members, but the Prime Minister presides over its
meetings and is the head of the Council.
Hence, option B is correct.

Q72. Consider the following statements:


Statement-I:
The Earth cools much faster during a clear night than during a cloudy night.
Statement-II:
Clouds act like blankets which prevent heat energy stored in the air and soil from leaving the Earth in the form of infrared
radiation.
Which one of the following is correct in respect of the above statements?
A. Both Statement-I and Statement-II are correct and Statement-II is the correct explanation for Statement-I.
B. Both Statement-I and Statement-II are correct and Statement-II is not the correct explanation for Statement-I.
C. Statement-I is correct but Statement-II is incorrect.
D. Statement-I is incorrect but Statement-II is correct.

2nd Floor, 45 Pusa Road, Opp. Metro Pillar 128, Karol Bagh, New Delhi-110005
Ph: 08045248491, 7041021151 | Email: students@levelupias.com
Answer: A
Explanation:
 Statement-I is correct: As the Earth rotates, the side facing away from the sun experiences nighttime. With no
direct sunlight, the Earth's surface starts to lose heat. The Earth loses heat primarily through infrared radiation,
a type of invisible light. This radiation travels outward from the warm Earth towards the colder space. When the
sky is clear, there are no clouds to obstruct this radiation. The Earth's surface can radiate heat directly towards
space, leading to a faster rate of cooling.
 Statement-II is correct: Clouds are composed of tiny water droplets or ice crystals suspended in the atmosphere.
They have the ability to reflect sunlight back into space, which helps to cool the Earth's surface by reducing the
amount of solar radiation that reaches the surface.
o Additionally, clouds absorb and re-emit infrared radiation emitted by the Earth's surface. This process
acts as an insulating layer, trapping heat and preventing it from escaping into space. In this way, clouds
act like a blanket, keeping the Earth's surface warmer than it would be without them.
Hence, option A is correct.

Q73. Consider the following statements with reference to ‘Mantle’:


1. The lower portion of the mantle is called asthenosphere.
2. It is the main source of magma that comes to the surface during volcanic eruptions.
3. The crust and the upper-most part of the mantle together are called lithosphere.
Which of the statements given above is/are correct?
A. 1 and 2 only
B. 2 only
C. 1 and 3 only
D. 2 and 3
Answer: D
Explanation:
 Statement 1 is not correct: The asthenosphere is the upper portion of the mantle. The asthenosphere is a region
of the upper mantle where rock behaves in a plastic, or semi-fluid, manner. This behavior allows the lithospheric
plates to move on the more fluid asthenosphere.
 Statement 2 is correct: The mantle is the main source of magma that finds its way to the surface during volcanic
eruptions. Magma is formed deep within the mantle through processes such as melting due to high temperature
and pressure, and it then rises towards the surface through cracks and weaknesses in the Earth's crust. When
magma reaches the surface, it erupts as lava, forming volcanoes.
 Statement 3 is correct: The lithosphere consists of the crust and the uppermost part of the mantle. It is the rigid
outer layer of the Earth that includes the crust and the solid uppermost part of the mantle. The lithosphere is
divided into tectonic plates, which move and interact with each other over the asthenosphere.
Hence, option D is correct.

2nd Floor, 45 Pusa Road, Opp. Metro Pillar 128, Karol Bagh, New Delhi-110005
Ph: 08045248491, 7041021151 | Email: students@levelupias.com
Q74. Consider the following plateaus:
1. Hazaribagh Plateau
2. Palamu Plateau
3. Ranchi Plateau
4. Malwa Plateau
How many of the above is/are part of the Peninsular India?
A. Only one
B. Only two
C. Only three
D. All four
Answer: D
Explanation:
 The Peninsular India is made up of a series of patland plateaus such as the Hazaribagh plateau, the Palamu
plateau, the Ranchi plateau, the Malwa plateau, the Coimbatore plateau and the Karnataka plateau, etc.
 Hazaribagh Plateau is located in the state of Jharkhand. It is characterized by its relatively flat terrain with low
hills and valleys. It is primarily composed of sedimentary rocks.
 Palamu Plateau is an extension of the Chota Nagpur Plateau. It is known for its rugged topography, dissected by
numerous rivers and streams.
 Ranchi Plateau encompasses the city of Ranchi. It is characterized by its undulating terrain, with numerous hills
and valleys.
 Malwa Plateau is located in the western part of India, primarily in the state of Madhya Pradesh, with extensions
into Rajasthan and Gujarat. It is a relatively flat region with low hills and shallow valleys.
Hence, option D is correct.

Q75. Consider the following statements:


1. A negative Growth Rate-Interest rate differential keeps the government debt sustainable.
2. Capex led growth will enable India to keep the growth interest differential positive in long run.
3. In India, Growth Rate-Interest rate differential has historically been generally positive.
Which of the above statements is/are correct?
A. 1 only
B. 1 and 2 only
C. 2 and 3 only
D. 1, 2 and 3
Answer: C
Explanation:

2nd Floor, 45 Pusa Road, Opp. Metro Pillar 128, Karol Bagh, New Delhi-110005
Ph: 08045248491, 7041021151 | Email: students@levelupias.com
 Statement 1 is not correct: A positive growth-interest differential means economic growth is higher than the
interest payment growth. This helps in keeping debt level sustainable.
 Statement 2 is correct: Capex led growth leads to higher economic growth in future and thus will keep growth-
interest differential positive.
 Statement 3 is correct: A positive growth-interest differential keeps the debt level sustainable. In India, this
differential has historically been positive.
Hence, option C is correct.

Q76. Consider the following statements with reference to the “Minimum Support Price (MSP)”:
1. It is the price at which the government procures certain crops from the farmers to safeguard them against any sharp
fall in the prices.
2. It is based on the recommendations of the Commission for Agricultural Costs and Prices (CACP).
3. The Government is legally mandated to declare the MSP before each cropping season.
How many of the above statements is/are correct?
A. Only one
B. Only two
C. All three
D. None
Answer: B
Explanation:
 Statement 1 is correct: The Minimum Support Price (MSP) is a guaranteed price offered by the government to
farmers for certain crops. If the market price falls below the MSP, the government agrees to purchase the crops
at the MSP, protecting farmers from financial losses. It is an integral component of the Agriculture Price Policy
and it strives to ensure support prices to the farmers and affordable prices to the consumers.
 Statement 2 is correct: The MSPs are announced by the Government of India at the beginning of the sowing
season for certain crops, on the basis of the recommendations of the Commission for Agricultural Costs and
Prices (CACP). CACP considers various factors like the cost of production, market prices, and the overall
agricultural situation. The CACP then submits its recommendations to the government, which makes the final
decision on the MSP.
 Statement 3 is not correct: The concept of MSPs first began in 1966 with the Green Revolution. Though, as of
April-2024, government is not legally mandated to do so. The government currently announces MSPs for 25
major agricultural commodities each year, including 14 crops of the Kharif season and 7 crops of the Rabi season.
Hence, option B is correct.

2nd Floor, 45 Pusa Road, Opp. Metro Pillar 128, Karol Bagh, New Delhi-110005
Ph: 08045248491, 7041021151 | Email: students@levelupias.com
Q77. National Monetization Pipeline was in news recently. It refers to:
A. A plan of getting receipt for government through monetization of brownfield infrastructure assets.
B. A plan of investment by government in dilapidated brownfield infrastructure assets.
C. A plan of getting receipts for government through monetization of new greenfield infrastructure assets.
D. A plan of investment by government in dilapidated greenfield infrastructure assets.
Answer: A
Explanation:
Budget 2021-22: Monetization of assets are very important for new infrastructure construction. A "National
Monetization Pipeline" of potential brownfield infrastructure assets was launched. An asset monetization dashboard
was created for tracking the progress and to provide visibility to investors.
Some important measures in the direction of Monetization are:
 NHAI and PGCIL each have sponsored one InvIT that will attract international and domestic institutional investors.
 Five operational roads with an estimated enterprise value of ` 5,000 crores are being transferred to the NHAI
InvIT.
 Similarly, transmission assets of a value of `7,000 crores will be transferred to the PGCIL InvIT.
 Railways will monetize Dedicated Freight Corridor assets for operations and maintenance, after commissioning.
 The next lot of Airports will be monetized for operations and management concession.
 Other core infrastructure assets that will be rolled out under the Asset Monetization Programme are:
o NHAI Operational Toll Roads
o Transmission Assets of PGCIL
o Oil and Gas Pipelines of GAIL, IOCL and HPCL
o AAI Airports in Tier II and III cities
o Other Railway Infrastructure Assets
o Warehousing Assets of CPSEs such as Central Warehousing Corporation and NAFED among others and
o Sports Stadiums
Hence, option A is correct.

Q78. Consider the following statements about the National Green Hydrogen Mission:
1. The mission has been launched by Ministry of Environment Forest and Climate Change.
2. It aims to increase the Green Hydrogen Production to at least 50 million metric tonnes by 2030.
3. It will contribute to reduction in fossil fuel import dependency in the country.
Which of the statements given above is/are correct?
A. 1 only
B. 1 and 2 only
C. 2 and 3 only
D. 3 only
Answer: D
Explanation:

2nd Floor, 45 Pusa Road, Opp. Metro Pillar 128, Karol Bagh, New Delhi-110005
Ph: 08045248491, 7041021151 | Email: students@levelupias.com
 Statement 1 is not correct: The National Green Hydrogen Mission has been launched by the Ministry of New and
Renewable Energy. The mission falls under the purview of renewable energy initiatives, focusing on promoting
green hydrogen production and usage to reduce carbon emissions.
 Statement 2 is not correct: The mission aims to develop green hydrogen production capacity to at least 5 million
metric tonnes per annum by 2030. The target is to establish a significant capacity for green hydrogen production
within the specified timeframe, contributing to India's renewable energy goals and reducing dependence on fossil
fuels.
 Statement 3 is correct: One of the primary objectives of the National Green Hydrogen Mission is to reduce
dependency on imported fossil fuels. By promoting the production and usage of green hydrogen, derived from
renewable energy sources like solar and wind power, India aims to decrease its reliance on imported fossil fuels
for various sectors such as industry, transportation, and energy generation. This reduction in import dependency
aligns with broader energy security and sustainability goals.
Hence, option D is correct.

Q79. Consider the following statements about Potential GDP:


1. Potential GDP is the highest levels at which an economy can produce.
2. If the GDP is less than potential GDP, the central bank generally tends to go for an expansionary monetary policy.
Which of the above statements is/are correct?
A. 1 only
B. 2 only
C. Both 1 and 2
D. Neither 1 nor 2
Answer: B
Explanation:
 Statement 1 is not correct: GDP can go even higher than potential GDP. Cost of rising inflation could make an
economy temporarily produce more than its potential level of output.
 Statement 2 is correct: If Potential GDP < GDP: The economy is overproducing, leading to inflationary pressures.
Central banks tighten monetary policy (raise interest rates, reduce money supply) to control inflation.
o If Potential GDP > GDP: The economy is underproducing, indicating unused resources and room for
growth. Central banks implement expansionary monetary policy (lower interest rates, increase money
supply) to stimulate economic activity and reach potential output.
Hence, option B is correct.

Q80. Consider the following statements:


1. Easing conditions relating to external commercial borrowing
2. Encouraging Indian borrowers to issue rupee-denominated Masala Bonds
3. Following an expansionary monetary policy
4. Curbing imports of non-essential goods and promoting exports
How many of the above steps is/are NOT likely to be taken by the Government/RBI to curb rupee depreciation?

2nd Floor, 45 Pusa Road, Opp. Metro Pillar 128, Karol Bagh, New Delhi-110005
Ph: 08045248491, 7041021151 | Email: students@levelupias.com
A. Only one
B. Only two
C. Only three
D. All four
Answer: A
Explanation:
 Statement 1 is not correct: By relaxing restrictions or providing incentives for external commercial borrowing,
the government/RBI can encourage Indian companies to borrow funds from abroad. This can increase the inflow
of foreign currency into the country, which can help stabilize or strengthen the rupee. Hence, this step is most
likely to be taken.
 Statement 2 is not correct: Masala bonds are rupee-denominated bonds issued by Indian entities in overseas
markets. By encouraging the issuance of these bonds, the government/RBI can attract foreign investors who
want to invest in Indian assets without the risk of currency exchange fluctuations. This can help increase foreign
inflows and support the rupee. Hence, this step is also most likely to be taken.
 Statement 3 is correct: Following an expansionary monetary policy may lead to lower interest rates thereby
increasing the inflation with higher imports through higher spending of the government and therefore further
slide of rupee takes place. Hence, this step is not likely to be taken.
 Statement 4 is not correct: Curbing imports of non-essential goods can help reduce the trade deficit and
reducing demand for foreign currency. Also, by promoting exports, the government/RBI can increase the inflow
of foreign currency, which can help support the rupee. Hence, this step is also most likely to be taken.
Hence, option A is correct.

Q81. Consider the following pairs:

GI Tag State

1. Lanji Saura painting Odisha

2. Mircha Rice Bihar

3. Dhenkanal Magji Madhya Pradesh

4. Kachchhi Kharek Maharashtra

Which of the above pairs is/are correctly matched?


A. 1 and 3 only
B. 2 and 4 only
C. 3 and 4 only
D. 1 and 2 only
Answer: D
Explanation:

2nd Floor, 45 Pusa Road, Opp. Metro Pillar 128, Karol Bagh, New Delhi-110005
Ph: 08045248491, 7041021151 | Email: students@levelupias.com
 Pair 1 is correct: Lanji Saura painting belongs to the Lanjia Saura community, a PVTG largely residing in the
Rayagada district. These paintings are in the form of exterior murals painted on the mud walls of homes. White
paintings figure over a crimson-maroon background.
 Pair 2 is correct: Mircha Rice of West Champaran (Bihar) has been awarded the GI tag. The size and shape of the
grain appear like that of black pepper; hence it is known as Mircha rice. The rice flakes of this rice have a unique
aroma that makes it different.
 Pair 3 is not correct: Dhenkanal Magji is a type of sweet made from cheese from buffalo milk, with distinct
characteristics in terms of appearance, taste, flavour, shape, and size. It also has unique nutritional values that
distinguish it from other cheese-based sweets. It belongs to Odisha (not Madhya Pradesh).
 Pair 4 is not correct: Kachchhi Kharek is an indigenous variety of dates of Kutch, has become the second fruit of
Gujarat (not Maharashtra) to get a geographical indication (GI) tag. Dates grown in Kutch are harvested at the
khalal stage, the stage when fruits have matured, accumulated sucrose, and have turned red or yellow but are
still crisp. In other countries, they are allowed to ripen further till they become soft and dark brown or black in
colour.
Hence, option D is correct.

Q82. Consider the following statements with reference to the Indian Star Tortoise:
1. The Indian Star Tortoise is endemic to the mountain ranges of the Western Ghats in south-western India only.
2. The Indian Star Tortoise is classified as ‘Endangered’ in the Red List of the International Union for Conservation of
Nature.
3. Loss of genetic diversity is among the major threats faced by the species.
How many of the above statements is/are correct?
A. Only one
B. Only two
C. All three
D. None
Answer: A
Explanation:
 Statement 1 is not correct: The Indian Star Tortoise is endemic to Indian subcontinent. It is found in three
separate parts of the Indian subcontinent: Western India and the extreme South-east of Pakistan (e.g., Gujarat,
Uttar Pradesh, and Rajasthan in India, as well as the Thar Desert in Pakistan), in South Eastern India (Kerala,
Karnataka, and Tamil Nadu), and in Sri Lanka. While part of its range overlaps with the Western Ghats, it is not
limited to this mountain range.
 Statement 2 is not correct: The Indian Star Tortoise is classified as "Vulnerable" on the IUCN Red List. It is placed
in Schedule IV of Wildlife (Protection) Act, 1972 and Appendix 1 of CITES.
 Statement 3 is correct: Loss of genetic diversity is one of the major threats faced by the Indian Star Tortoise. This
loss occurs due to subsequent hybridization between different populations over the years. Other threats include
habitat fragmentation due to urbanization and agriculture, illegal collection for the pet trade, and accidental
killings.

2nd Floor, 45 Pusa Road, Opp. Metro Pillar 128, Karol Bagh, New Delhi-110005
Ph: 08045248491, 7041021151 | Email: students@levelupias.com
Hence, option A is correct.

Q83. Consider the following statements with reference to the Wildlife (Protection) Amendment Act, 2022:
1. One of the aims of the amendment is to incorporate some of the provisions of the Convention on International Trade
in Endangered Species of Wild Fauna and Flora (CITES) in the Act.
2. It has a provision for the constitution of a Standing Committee to exercise powers and duties delegated by the State
Wildlife Boards.
3. The Act strictly bans the use of elephants for ‘religious or any other purpose’.
How many of the above statements is/are correct?
A. Only one
B. Only two
C. All three
D. None
Answer: B
Explanation:
 Statement 1 is correct: The Wildlife (Protection) Amendment Act, 2022, aims to enhance protection for wildlife
species by incorporating provisions from the Convention on International Trade in Endangered Species of Wild
Fauna and Flora (CITES) into the Wildlife (Protection) Act, 1972. This means that the Act will now regulate and
protect species listed under the CITES Appendices more effectively by aligning its regulations with the
international standards set by CITES.
 Statement 2 is correct: The Act introduces a provision for the establishment of a Standing Committee. This
committee is tasked with exercising powers and duties delegated by the State Wildlife Boards. Essentially, this
committee serves as a body that can handle specific responsibilities related to wildlife protection and
management at the state level, working in coordination with the State Wildlife Boards.
 Statement 3 is not correct: One of the significant amendments made by the Act is to Section 43. Previously,
Section 43 might have had restrictions or regulations regarding the use of certain animals, including elephants.
However, the Amendment Act now permits the use of elephants, categorized as Schedule animals, for 'religious
or any other purpose'. This indicates a departure from strict bans on the use of elephants for such purposes,
potentially allowing for cultural or religious practices involving elephants to continue under certain conditions.
Hence, option B is correct.

2nd Floor, 45 Pusa Road, Opp. Metro Pillar 128, Karol Bagh, New Delhi-110005
Ph: 08045248491, 7041021151 | Email: students@levelupias.com
Q84. Consider the following pairs:
River Originating/Flowing
Biosphere Reserve
through the Reserve

1. Nanda Devi Mandakini

2. Panna Betwa

3. Achanakmar-
Son
Amarkantak

4. Agasthyamalai Neyyar

Which of the above pairs is/are correct?


A. 1 and 2 only
B. 2 and 4 only
C. 2 and 3 only
D. 3 and 4 only
Answer: D
Explanation:
 Pair 1 is not correct: Nanda Devi Biosphere Reserve is located in Uttarakhand. It comprises the catchment
area of the Rishi Ganga, an eastern tributary of Dhauli Ganga which flows into the Alaknanda River at
Joshimath.
o There are two core zones, namely Nanda Devi National Park and Valley of Flower National Park. These
parks include the catchment of Alaknanda River and its tributaries including Rishi Ganga, Dhauli
Ganga, Pushwapati and Khiro Ganga.
 Pair 2 is not correct: River Ken (not Betwa) flows through Panna Biosphere Reserve, which lies in the State
of Madhya Pradesh.
 Pair 3 is correct: Achanakmar Amarkantak Biosphere Reserve lies in the state of Chhattisgarh, Madhya
Pradesh. It is the source of three major river systems: Narmada, Johilla and Son.
 Pair 4 is correct: Agasthyamalai Biosphere Reserve is located in Kerala and Tamil Nadu. Neyyar river flows
through it.
Hence, option D is correct.

2nd Floor, 45 Pusa Road, Opp. Metro Pillar 128, Karol Bagh, New Delhi-110005
Ph: 08045248491, 7041021151 | Email: students@levelupias.com
Q85. Consider the following statements:
Statement-I:
Native ecosystems that have undergone human induced disturbance are often more prone to alien invasions than
undisturbed ecosystems.
Statement-II:
Ecosystems that are invaded by alien species might lack the natural predators and competitors found in their native
environment, which would typically regulate their populations.
Which one of the following is correct in respect of the above statements?
A. Both Statement-I and Statement-II are correct and Statement-II is the correct explanation for Statement-I.
B. Both Statement-I and Statement-II are correct and Statement-II is not the correct explanation for Statement-I.
C. Statement-I is correct but Statement-II is incorrect.
D. Statement-I is incorrect but Statement-II is correct.
Answer: B
Explanation:
 Statement-I is correct: Human-induced disturbances, such as habitat destruction, pollution, and introduction of
non-native species, can disrupt the balance of native ecosystems. These disturbances often reduce the resilience
of the ecosystem, making it more susceptible to invasion by alien species.
o Disturbed ecosystems often have niches (ecological roles) left empty due to the loss of native species.
These empty niches can be easily filled by invasive species that don't face the competition or predation
they would encounter in their native habitat.
 Statement-II is correct: When species are introduced to ecosystems outside their native range, they may
encounter a lack of natural enemies, such as predators or competitors, that would normally keep their
populations in check. Without these natural controls, these species can experience rapid population growth and
spread, potentially outcompeting native species for resources and disrupting ecological communities.
 While lack of natural controls can contribute to successful invasion, it's not the sole reason. Even with predators
or competitors present, a disturbed ecosystem might be more vulnerable due to:
o Reduced competition for resources
o Disruption of natural defense mechanisms (e.g., plants less able to resist herbivores)
o Introduction of pathways for invasion (e.g., disturbed soil for seed dispersal)
 Therefore, both statements are correct, but Statement-II offers a partial explanation for why disturbed
ecosystems are more prone to invasions. Other factors also play a role.
Hence, option B is correct.

2nd Floor, 45 Pusa Road, Opp. Metro Pillar 128, Karol Bagh, New Delhi-110005
Ph: 08045248491, 7041021151 | Email: students@levelupias.com
Q86. Why is CNG considered as a better fuel than Diesel?
1. CNG burns more efficiently than diesel.
2. Easy access to the supply of CNG.
3. CNG is cheaper than diesel.
4. CNG cannot be siphoned off by thieves and adulterated like diesel easily.
Select the correct answer using the code given below:
A. 2 and 4 only
B. 1, 3 and 4 only
C. 1 and 3 only
D. 1, 2, 3 and 4
Answer: B
Explanation:
 Statement 1 is correct: CNG has a higher-octane rating than diesel. Octane rating indicates a fuel's resistance to
knocking (premature ignition). During combustion, a higher-octane rating allows for a more controlled burn,
resulting in less wasted energy and potentially better fuel economy.
 Statement 2 is not correct: The main problem with switching over to CNG is the difficulty of laying down pipelines
to deliver CNG through distribution points/pumps and ensuring uninterrupted supply. CNG infrastructure is not
as widespread as diesel stations, so refuelling can be less convenient.
 Statement 3 is correct: In general, CNG tends to be cheaper per gallon compared to diesel. This can lead to
significant cost savings on fuel, especially for high-mileage drivers. However, this cost advantage can vary
depending on factors such as local taxation policies, production and transportation costs, and government
subsidies or incentives aimed at promoting the use of cleaner fuels.
 Statement 4 is correct: Unlike liquid fuels such as diesel, which can be siphoned off from tanks and adulterated
with lower-quality substances or additives, CNG is stored in specially designed high-pressure cylinders within
the vehicle. These cylinders are difficult to tamper with and not easily accessible, making them theft-proof.
Hence, option B is correct.

Q87. How many of the following are classified as ‘Insects’?


1. Dragonflies
2. Moths
3. Butterflies
Select the correct answer using the code given below.
A. Only one
B. Only two
C. All three

2nd Floor, 45 Pusa Road, Opp. Metro Pillar 128, Karol Bagh, New Delhi-110005
Ph: 08045248491, 7041021151 | Email: students@levelupias.com
D. None
Answer: C
Explanation:
 Statement 1 is correct: A dragonfly is a flying insect belonging to the infraorder Anisoptera below the order
Odonata.
 Statement 2 is correct: Moths are group of insects that include all members of the order Lepidoptera that are
not butterflies.
 Statement 3 is correct: Butterflies are insects that have large, often brightly coloured wings, and a conspicuous,
fluttering flight.
Hence, option C is correct.

Q88. Consider the following statements with reference to the Air (Prevention and Control of Pollution) Act, 1981:
1. The Act was to implement the decisions taken at the United Nations Conference on the Human Environment, held at
Stockholm in 1972.
2. The Central Pollution Control Board was constituted under the Act.
Which of the statements given above is/are correct?
A. 1 only
B. 2 only
C. Both 1 and 2
D. Neither 1 nor 2
Answer: A
Explanation:
 Statement 1 is correct: Air (Prevention and Control of Pollution) Act, 1981 aimed to address the issue of air
pollution in India and was enacted to fulfill the commitments made by the country at the United Nations
Conference on the Human Environment held in Stockholm in 1972. The conference emphasized the need for
nations to take steps to preserve the quality of air and control air pollution, among other environmental concerns.
Therefore, the Indian government passed this act to align with the decisions and goals set forth in the conference.
 Statement 2 is not correct: The Central Pollution Control Board (CPCB) was established under the Water
(Prevention and Control of Pollution) Act, 1974.
o While the Air (Prevention and Control of Pollution) Act, 1981, does empower the CPCB to exercise
certain powers and functions related to the prevention and control of air pollution. The CPCB plays a
significant role in coordinating efforts to control air pollution at the national level.
Hence, option A is correct.

2nd Floor, 45 Pusa Road, Opp. Metro Pillar 128, Karol Bagh, New Delhi-110005
Ph: 08045248491, 7041021151 | Email: students@levelupias.com
Q89. Consider the following statements about NOMA disease:
1. It is a bacterial disease and primarily affects children between 2 to 6 years of age.
2. It is a contagious disease and spread from human to human by touch.
3. It may cause severe disfigurement for survivors.
4. It has recently been included in the list of Neglected Tropical Disease (NTD) by the World Health Organization.
How many of the above statements is/are correct?
A. Only one
B. Only two
C. Only three
D. All four
Answer: C
Explanation:
 Statement 1 is correct: Evidence indicate that NOMA is caused by bacteria found in the mouth. It is a severe
gangrenous disease of the mouth and face. It primarily affects young children (between the ages of 2 years to 6
years) in regions of extreme poverty.
 Statement 2 is not correct: It is not a contagious disease and cannot be spread directly from person to person. It
tends to strike when the body's immune system is weak.
 Statement 3 is correct: It can be fatal in some cases and may also cause severe disfigurement. The rapid
destruction of tissues in the mouth and face can leave permanent scarring and functional impairments. These
disfigurements can have profound physical, psychological, and social impacts on the affected individuals, often
leading to stigmatization and social isolation.
 Statement 4 is correct: 'Noma' is the latest addition to WHO's list of neglected tropical diseases.
o Significance of including NOMA in the NTD's list of WHO:
 Amplify global awareness.
 Catalyse research, stimulate funding and boost efforts to control the disease through
multisectoral and multi-pronged approaches.
Hence, option C is correct.

Q90. Consider the following statements:


Statement-I:
SpaceX's Inspiration4 mission took civilians to an orbital mission to a height of more than 500 km above earth.
Statement-II:
In sub-orbital flights weightlessness can't be felt.
Which one of the following is correct in respect of the above statements?
A. Both Statement-I and Statement-II are correct and Statement-II is the correct explanation for Statement-I.
B. Both Statement-I and Statement-II are correct and Statement-II is not the correct explanation for Statement-I.
C. Statement-I is correct but Statement-II is incorrect.

2nd Floor, 45 Pusa Road, Opp. Metro Pillar 128, Karol Bagh, New Delhi-110005
Ph: 08045248491, 7041021151 | Email: students@levelupias.com
D. Statement-I is incorrect but Statement-II is correct.
Answer: C
Explanation:
 Statement-I is correct: In space tourism civilians are taken to orbital flight around earth. But it has to be noted
the space tourism also happens in suborbital flights (for e.g. Virgin Galactica's flight or Blue Origin's flight).
o Example - SpaceX’s inspiration4 took civilians to an orbital flight. Whereas, Blue Origin’s flight to space
have been suborbital.
 Statement-II is not correct: Weightless will occur when the spacecraft/vehicle is moving only with the force of
gravity. Even in case of sub-orbital flights, weightlessness may occur at the time when the spacecraft is falling
back on earth.
o Example - In case of when Blue Origin's New Shepherd rocket takes scientists to a suborbital flight, while
coming back on earth, they experience weightlessness.
Hence, option C is correct.

Q91. Consider the following statements with reference to the Fungi Kingdom:
1. All organisms in this kingdom are heterotrophic in nature.
2. Some of the organisms in this kingdom are useful for making medicines.
3. Organisms in this kingdom are unicellular only.
4. Fungi can be found in air, water, soil, and on animals and plants.
How many of the above statements is/are correct?
A. Only one
B. Only two
C. Only three
D. All four
Answer: C
Explanation:
 Statement 1 is correct: Fungi are heterotrophic, meaning they cannot produce their own food and rely on
external sources of organic carbon for their nutrition. They obtain nutrients by secreting enzymes that break
down complex organic molecules into simpler compounds, which they can then absorb.
 Statement 2 is correct: Many fungi are used in the production of antibiotics, including Penicillium, which
produces penicillin. Other fungi are used in the production of various drugs, such as statins (used to lower
cholesterol) and immunosuppressants (used in organ transplantation).
 Statement 3 is not correct: While some fungi, such as yeasts, are unicellular, most fungi are multicellular and
form complex, filamentous structures called hyphae. The network of hyphae is known as mycelium, which is the
main body of the fungus.
 Statement 4 is correct: Fungi are cosmopolitan and can be found in diverse habitats, including air, water, soil,
and on the surfaces of plants and animals. They play important roles in ecosystems as decomposers, helping to
break down organic matter.

2nd Floor, 45 Pusa Road, Opp. Metro Pillar 128, Karol Bagh, New Delhi-110005
Ph: 08045248491, 7041021151 | Email: students@levelupias.com
Hence, option C is correct.

Q92. Consider the following statements:


Statement-I:
Meats, fish, poultry etc are not assigned a Glycaemic Index (GI).
Statement-II:
Only food containing carbohydrates are assigned glycaemic Index.
Which one of the following is correct in respect of the above statements?
A. Both Statement-I and Statement-II are correct and Statement-II is the correct explanation for Statement-I.
B. Both Statement-I and Statement-II are correct and Statement-II is not the correct explanation for Statement-I.
C. Statement-I is correct but Statement-II is incorrect.
D. Statement-I is incorrect but Statement-II is correct.
Answer: A
Explanation:
 Statement-I is correct: Meats, fish, and poultry are primarily composed of protein and fat, with little to no
carbohydrates. The glycemic index ranks foods based on their impact on blood sugar levels, and carbohydrates
are the main dietary component that affects blood sugar. Since meats have negligible carbohydrates, they don't
cause a rise in blood sugar and therefore aren't assigned a glycemic index value.
 Statement-II is correct: The glycemic index system applies to carbohydrate-containing foods because these
carbohydrates are broken down into sugars in the body, leading to blood sugar fluctuations. Foods rich in protein
and fat, like meats, have minimal effects on blood sugar and thus aren't included in the glycemic index ranking.

Hence, option A is correct.

Q93. Consider the following statements:


Statement-I:
India's newly planned spaceport at Kulasekarripattinam will allow small satellite launch vehicles (SSLVs) to carry higher
payload when compared to launches of SSLVs from spaceport at Satish Dhawan Space Centre, Sriharikota.
Statement-II:
The launches from Kulasekarripattinam will not require the Dogleg Manoeuvre to avoid flying over Sri Lanka, thus saving
fuel and allowing for higher payload.
Which one of the following is correct in respect of the above statements?
A. Both Statement-I and Statement-II are correct and Statement-II is the correct explanation for Statement-I.
B. Both Statement-I and Statement-II are correct and Statement-II is not the correct explanation for Statement-I.
C. Statement-I is correct but Statement-II is incorrect.
D. Statement-I is incorrect but Statement-II is correct.
Answer: A

2nd Floor, 45 Pusa Road, Opp. Metro Pillar 128, Karol Bagh, New Delhi-110005
Ph: 08045248491, 7041021151 | Email: students@levelupias.com
Explanation:
 Statement-I is correct: In Feb 2024, PM Modi laid down the
foundation stone for India’s second spaceport at
Kulasekharapattinam, a coastal village in TN’s Thoothukudi
district on 28th Feb 2024.
o The Kulasekharapattinam space port will allow a direct
southward and smaller trajectory for the light weight
SSLVs carrying less fuel. It is because
Kulasekharapattinam is located several kms to the
west of Colombo. It will enhance the payload capacity.

 Statement-II is correct: Problem with launching small


satellites from Sriharikota: Dogleg Maneuver takes extra fuel
and reduces the payload capacity.
o Polar satellites launched from Sriharikota spaceport of
south India frequently use this maneuver to avoid
flying over Sri Lanka. Rockets make a steep 40-degree
arc in order to bypass the city of Colombo. For larger
satellites, fuel required for this maneuver is insignificant
compared to the total fuel. However, this is very
inefficient for smaller rockets.
Hence, option A is correct.

Q94. Consider the following statements with reference to reduced height (Rht) or alternative dwarfing genes:
1. Rht 14 and Rht 18 are two alternative dwarfing genes of rice which promote better seedling vigour.
2. These genes can reduce crop stubble burning incidences under the rice-wheat cropping system. .
3. These genes are mapped by Agharkar Research Institute.
4. They shorten the coleoptile, hindering emergence under deep sowing of seeds.
Which of the statements given above is/are correct?
A. 1, 2 and 4 only
B. 1, 3 and 4 only
C. 1, 2 and 3 only
D. 2 and 3 only
Answer: D
Explanation:
 Statement 1 is not correct: Rht 14 and Rht 18 are alternative dwarfing genes found in wheat (not rice). These
genes have been mapped on chromosome 6A in a durum variety of wheat by scientists at the Agharkar Research
Institute (ARI).
 Statement 2 is correct: The mapping of these alternative dwarfing genes in wheat by ARI is significant because it
can help reduce stubble burning incidences in the rice-wheat cropping system. Farmers often burn leftover rice

2nd Floor, 45 Pusa Road, Opp. Metro Pillar 128, Karol Bagh, New Delhi-110005
Ph: 08045248491, 7041021151 | Email: students@levelupias.com
residues to prepare their fields for sowing wheat, leading to air pollution. The alternative dwarfing genes allow
wheat to be sown under rice stubble-retained conditions, eliminating the need for stubble burning.
 Statement 3 is correct: Pune based Agharkar Research Institute (ARI) mapped the dwarfing genes Rht14 and
Rht18 on chromosome 6A in a durum variety of wheat. They also developed DNA-based markers for a better
selection of these genes in wheat breeding lines.
 Statement 4 is not correct: The coleoptile, a protective sheath for the seedling shoot tip, plays a crucial role in
pushing through the soil. Unlike some other dwarfing genes, Rht14 and Rht18 do not affect coleoptile length and
seedling shoot length. This characteristic makes them suitable for deep sowing conditions, as they retain early
vigour in wheat seedlings.
Hence, option D is correct.

Q95. Consider the following statements about the Jurisdiction of International Court of Justice (ICJ):
1. ICJ can give binding ruling on disputes between the provincial units of any member state.
2. A non-state interest cannot be the subject of proceedings in the ICJ.
3. ICJ's Advisory jurisdiction is confined to specialized UN bodies and agencies.
4. ICJ's advisory jurisdiction is only consultative in character.
Which of the statements given above is/are correct?
A. 1 and 2 only
B. 2 and 3 only
C. 3 and 4 only
D. 1, 2, 3 and 4
Answer: C
Explanation:
 Statement 1 is not correct: ICJ produces a binding ruling between states. Individual, corporations, parts of
federal states, NGOs, UN organs and self-determination groups are excluded from direct participation in cases.
 Statement 2 is not correct: A non-state actor interest can be the subject of proceedings if one state brings the
case against another.
 Statement 3 and 4 are correct: Advisory Opinion is a function of court only to specialized UN bodies and
agencies. Advisory Opinion is a means by which UN agencies could seek the court's help in deciding complex
legal issues that might fall under their respective mandates.
o This advisory jurisdiction is consultative in character but is very influential and widely respected.
Hence, option C is correct.

2nd Floor, 45 Pusa Road, Opp. Metro Pillar 128, Karol Bagh, New Delhi-110005
Ph: 08045248491, 7041021151 | Email: students@levelupias.com
Q96. Which of the following is NOT correct about Gulf Cooperation Council (GCC)?
A. Iraq is the only Arab country bordering Persian Gulf that is not part of GCC.
B. All the countries of GCC are Monarchies.
C. It is an intergovernmental organization formed in 1960 to regulate the supply of crude oil.
D. None of the above.
Answer: C
Explanation:
 Option (a) is correct: The Gulf Cooperation Council (GCC) consists of six member states: Bahrain, Kuwait, Oman,
Qatar, Saudi Arabia, and the United Arab Emirates (UAE).
o Iraq, despite being an Arab country with a coastline on the Persian Gulf, is not part of the GCC.
 Option (b) is correct: Each member state of the GCC is governed by a monarchy, either an absolute monarchy or
a constitutional monarchy. The rulers of these monarchies hold varying degrees of power within their respective
countries.
 Option (c) is not correct: The Gulf Cooperation Council (GCC) was established on May 25, 1981, in Abu Dhabi,
United Arab Emirates. Its primary objectives include enhancing coordination, integration, and cooperation
among its member states in various fields such as economics, politics, and security.
o OPEC was formed in 1960 to regulate the supply of Crude oil.
Hence, option C is correct.

Q97. What is the correct sequence of occurrence of the following Lakes from North to South?
1. Lake Turkana
2. Lake Victoria
3. Lake Tanganyika
4. Lake Malawi

2nd Floor, 45 Pusa Road, Opp. Metro Pillar 128, Karol Bagh, New Delhi-110005
Ph: 08045248491, 7041021151 | Email: students@levelupias.com
Select the correct answer using the code given below:
A. 4-2-1-3
B. 1-2-3-4
C. 3-4-2-1
D. 4-3-2-1
Answer: B
Explanation:
The correct sequence is 1-2-3-4 (Lake Turkana - Lake
Victoria - Lake Tanganyika - Lake Malawi).
 The African Great Lakes are a series of lakes which
are part of the Rift Valley lakes and are located in
and around the East African Rift. The major lakes
are Lake Victoria, Lake Tanganyika, and Lake
Malawi.
 The African great lakes consist of seven lakes
These are Lake Victoria, Tanganyika, Malawi,
Turkana, Albert, Kivu and Edward in order of
size.
Hence, option B is correct.

Q98. Consider the following countries:


1. Poland
2. Slovakia
3. Switzerland
4. Belarus
5. Hungary
How many of the above-mentioned countries are landlocked?

2nd Floor, 45 Pusa Road, Opp. Metro Pillar 128, Karol Bagh, New Delhi-110005
Ph: 08045248491, 7041021151 | Email: students@levelupias.com
A. Only two
B. Only three
C. Only four
D. All five
Answer: C
Explanation:
A landlocked country is a country that does
not have territory connected to an ocean or
whose coastlines lie solely on endorheic
basins.
 Slovakia, Switzerland, Belarus
and Hungary are amongst the
landlocked countries of Europe.
o Poland is not
landlocked. It has a
coastline along the
Baltic Sea in the north
Hence, option C is correct.

Q99. Consider the following statements with reference to Mission Utkarsh:


1. It aims to improve the nutritional status of adolescent girls in all the aspirational districts.
2. The project is coordinated by the Central Council for Research in Ayurvedic Sciences (CCRAS).
3. The project's goal is to improve the health of anemic adolescent girls (14-18 years) by providing classical Ayurveda
medicines.
Which of the statements given above is/are correct?
A. 1 only
B. 1 and 2 only
C. 1, 2 and 3
D. 2 and 3 only
Answer: D
Explanation:
 Statement 1 is not correct: Mission Utkarsh is a project that aims to improve the nutritional status of adolescent
girls in five aspirational districts of five states.
o The five districts are Assam (Dhubri), Chhattisgarh (Bastar), Jharkhand (Paschimi Singhbhum),
Maharashtra (Gadchiroli), and Rajasthan (Dhaulpur).
 Statement 2 is correct: The project is coordinated by the Central Council for Research in Ayurvedic Sciences
(CCRAS) and involves 15 central ministries or departments.

2nd Floor, 45 Pusa Road, Opp. Metro Pillar 128, Karol Bagh, New Delhi-110005
Ph: 08045248491, 7041021151 | Email: students@levelupias.com
 Statement 3 is correct: The project's goal is to improve the health of anemic adolescent girls (14-18 years) by
providing classical Ayurveda medicines for three months. The project will cover approximately 10,000
Anganwadi Centers in the five districts and benefit over 94,000 adolescent girls.
Hence, option D is correct.

Q100. Consider the following statements about Mass Spectrometry:


1. It is used for measuring mass to charge ratio (m/z) of one or more molecules present in the sample.
2. It is used in radio-carbon dating to determine the amount of Carbon-14 left in the organic sample.
3. Gieger Counter used for radio-carbon dating initially also used Mass Spectrometry.
How many of the above statements are correct?
A. Only one
B. Only two
C. Only three
D. None of the above
Answer: B
Explanation:
 Statement 1 is correct: Mass spectrometry is an analytical tool useful for measuring the mass-to-charge
ratio (m/z) of one or more molecules present in a sample. These measurements can often be used to calculate
the exact molecular weight of the sample components as well. Typically, mass spectrometers can be used to
identify unknown compounds via molecular weight determination, to quantify known compounds, and to
determine structure and chemical properties of molecules.
 Statement 2 is correct: Accelerator mass spectrometry (AMS) is one of the most sensitive devices used to
determine amount of C-14 left in the organic sample.
o Mass spectrometry is used to isolate ions that have the same mass-to-charge ratio. They begin with a
sample – for e.g. a piece of bone – bombard it with electrons to ionize the atoms. Then they subject ions
to different physical conditions that cause them to separate according to their mass-to-charge ratio.
o Example - When deflected by electric or magnetic fields – Ions with different mass-to-charge ratios are
deflected to different extents.
 Statement 3 is not correct: It didn’t use mass spectrometry. Geiger Counter was used in 1940s when
radiocarbon dating began. It consists of a Geiger-Muller tube connected to some electronics that interpret and
display signals.
o The Geiger-Muller tube contains a noble gas, such as helium or neon, and a rod passing through the
centre. A high voltage is maintained between the tube’s inner surface and the rod. The gas is insulating,
so no current can pass between the two. But when energetic particles (including gamma radiation), such
as those emitted during radioactive decay, pass through the gas, they can energize electrons in the gas’s
atoms and produce an electric discharge. The persistent voltage could also encourage these electrons
to knock off electrons in more atoms, producing a bigger discharge (called the Townsend discharge).
This electric signal is relayed to the electronics, where, say, a light may come on in response, indicating
that radioactive decay is happening nearby.

2nd Floor, 45 Pusa Road, Opp. Metro Pillar 128, Karol Bagh, New Delhi-110005
Ph: 08045248491, 7041021151 | Email: students@levelupias.com
Hence, option B is correct.

2nd Floor, 45 Pusa Road, Opp. Metro Pillar 128, Karol Bagh, New Delhi-110005
Ph: 08045248491, 7041021151 | Email: students@levelupias.com

You might also like